Найти решение системы уравнений: Решение систем уравнений — Калькулятор Онлайн

Содержание

Решение системы линейных уравнений

Решение системы линейных уравнений

При помощи рассмотренного ранее инструмента Поиск решения можно также легко решать системы линейных равнений.

Системой n линейных уравнений с m неизвестными называется система вида:

где: aij и bi (i = 1, , m; b = 1, , n) – некоторые известные числа, x1, , xn – неизвестные.

В обозначении коэффициентов aij первый индекс i обозначает номер уравнения, а второй индекс j – номер неизвестного, при котором стоит этот коэффициент.

Коэффициенты при неизвестных часто записывают в виде матрицы, которая называется матрицей системы:

Числа, стоящие в правых частях уравнений,

b1, , bm называются свободными членами.

Совокупность n чисел c1, , cn называется решением данной системы уравнений, если каждое уравнение обращается в равенство после подстановки в него чисел c1, , cn вместо соответствующих неизвестных x1, , xn.

Наша задача найти решение системы. При этом могут возникнуть три ситуации:

  • Система может иметь единственное решение.
  • Система может иметь бесконечное множество решений.
  • И третий случай, когда система вообще не имеет решения.

Система линейных уравнений, имеющая хотя бы одно решение, называется совместной. В противном случае, т.е. если система не имеет решений, то она называется несовместной.

Существуют различные способы нахождения решений системы:

  • Матричный метод решения систем линейных уравнений.
  • Метод Крамера.
  • Метод Гаусса.

Создадим новый метод решения системы линейных уравнений, используя инструмент Поиск решения.

В качестве примера рассмотрим систему 3-х линейных уравнений с 3-мя неизвестными:

Коэффициенты при неизвестных и свободные члены запишем в виде таблицы:

В ячейках A7:C7 находятся будущие корни системы уравнений. Сначала присвоим им нулевые значения:

В ячейку D2 запишем формулу для вычисления свободного члена первого уравнения =СУММПРОИЗВ($A$7:$C$7;A2:C2). Эта функция перемножает соответствующие элементы заданных массивов одинаковой размерности и возвращает сумму произведений. Затем сделаем автозаполнение формулы для двух других ячеек в столбце

D:

Вот так выглядят наши таблички в режиме значений:

Наша задача добиться попарного совпадения значений в соседних ячейках столбцов D и E, изменяя значения в ячейках A7:C7. Для этого будем использовать Поиск решения (на вкладке Данные → в группе Анализ → кнопка Поиск решения):

В диалоговом окне
Поиск решения устанавливаем следующие параметры:
  1. Установить целевую ячейку: тут должно быть пусто (стираем все значения, если они там были).
  2. Равной: значению: 0.
  3. Изменяя ячейки: $A$7:$C$7 (подбираем корни системы уравнений).
  4. Ограничения: $D$2:$D$4=$E$2:$E$4.

Нажимаем кнопку Выполнить:

Оставляем пункт Сохранить найденное решение → кнопка OK.

Теперь в ячейках A7, B7 и С7 будут записаны точные значения корней системы уравнений:

В противном случае в окне Результаты поиска решения получим сообщение Поиск не может найти подходящего решения:

Система линейных уравнений с двумя переменными.

Методы решения систем уравнений.

Решением системы линейных уравнений двух переменных является любая упорядоченная пара, удовлетворяющая каждому уравнению независимо. Мы можем проверить решение, подставив значения в каждое уравнение, чтобы увидеть, удовлетворяет ли упорядоченная пара обоим уравнениям.

Как можно решить систему уравнений с двумя переменными?

Системы уравнений с двумя переменными можно решить методом подстановки:

 

 


 

Системы уравнений с двумя переменными можно решить методом сложения:

Пример. Решить систему методом сложения: \(\begin{equation*} \begin{cases} x-y-4=0 \\ 3x+y-8=0 \end{cases} \end{equation*}\).

Решение:

 


Система уравнений состоящее из двух переменных должно удовлетворять всем решениям одновременно. Система линейных уравнений из двух переменных рассматривается одновременно. Чтобы найти единственное решение системы линейных уравнений, мы должны найти численное значение для каждой переменной в системе, которая будет удовлетворять всем уравнениям системы одновременно. Некоторые линейные системы могут не иметь решения, и это  будет их решением, другие системы могут иметь бесконечное число решений. Для того чтобы линейная система имела единственное решение, должно быть не меньше уравнений, чем переменных. Тем не менее, это не гарантирует уникальное решение.

Выводы:

  • Система линейных уравнений из двух переменных решается совместно методом подстановки или методом сложения.
  • Чтобы найти решение системы линейных уравнений, мы должны найти численное значение для каждой переменной в системе, которая будет удовлетворять всем уравнениям в системе одновременно.
  • Для того чтобы линейная система имела единственное решение, должно быть не меньше уравнений, чем переменных.
  • Решить систему уравнений это значит найти численное значение для каждой переменной в системе либо доказать что решений нет.

 

Больше уроков и заданий по математике вместе с преподавателями нашей онлайн-школы «Альфа». Запишитесь на пробное занятие уже сейчас!

Запишитесь на бесплатное тестирование знаний!

Наши преподаватели

Оставить заявку

Репетитор по математике

Душанбинский государственный педагогический институт им.

Т. Г. Шевченко

Проведенных занятий:

Форма обучения:

Дистанционно (Скайп)

Репетитор по математике 5-9 классов.

Подготовка к ОГЭ. Я люблю математику и очень стараюсь привить эту любовь детям. Ведь без математики наша жизнь была бы скучной и однообразной, мы много бы не узнали об окружающем нас мире. К каждому ребенку нахожу индивидуальный подход. Объясняю доступно, легко и просто.

Оставить заявку

Репетитор по математике

Актюбинский педагогический техникум

Проведенных занятий:

Форма обучения:

Дистанционно (Скайп)

Репетитор 1-5 классов. Люблю работать с детьми и умею находить с ними общий язык. Математику люблю за точность и преподаю её так, как хотела бы, чтобы учили моего ребенка. При обучении настраиваю на позитивное восприятие всего нового и непонятного. Весело и интересно объясняю сложный материал. Со мной вашим детям будет легко, занимательно и познавательно. До встречи на уроках!

Оставить заявку

Репетитор по математике

Тобольская государственная социально-педагогическая академия

Проведенных занятий:

Форма обучения:

Дистанционно (Скайп)

Репетитор по английскому языку 9-11 классы. Подготовка к ОГЭ/ЕГЭ. Работаю по своей авторской методике, основанной на знании возрастной психологии. Многолетний опыт индивидуальной работы с учениками разных возрастов показал, что «необучаемых» детей почти не бывает! Язык дает мне возможность понимать и принимать культуру, менталитет разных стран, расширяет кругозор, помогает посмотреть на ситуацию с разных точек зрения. Уроки проходят в формате живого общения и постоянного психо-эмоциального контакта с учащимися. Избегаю «технического» метода в обучении (зубрежки, знания без умений, буквального оценивания) Более десятка учеников разных возрастов смогли повысить успеваемость в школе, чувствовать себя уверенно на уроках, стали лучше понимать грамматику и иностранную речь, ученикам выпускных классов за короткие сроки удалось сдать ОГЭ и ЕГЭ на высокие баллы, несмотря на то, что за «пробники» в школе баллы были очень низкие.

Математика 10 класс

  • — Индивидуальные занятия
  • — В любое удобное для вас время
  • — Бесплатное вводное занятие

Похожие статьи

Записаться на бесплатный урок

Решение уравнений в Excel методом итераций Крамера и Гаусса

В программе Excel имеется обширный инструментарий для решения различных видов уравнений разными методами.

Рассмотрим на примерах некоторые варианты решений.

Решение уравнений методом подбора параметров Excel

Инструмент «Подбор параметра» применяется в ситуации, когда известен результат, но неизвестны аргументы. Excel подбирает значения до тех пор, пока вычисление не даст нужный итог.

Путь к команде: «Данные» — «Работа с данными» — «Анализ «что-если»» — «Подбор параметра».

Рассмотрим на примере решение квадратного уравнения х2 + 3х + 2 = 0. Порядок нахождения корня средствами Excel:

  1. Введем в ячейку В2 формулу для нахождения значения функции. В качестве аргумента применим ссылку на ячейку В1.
  2. Открываем меню инструмента «Подбор параметра». В графе «Установить в ячейку» — ссылка на ячейку В2, где находится формула. В поле «Значение» вводим 0. Это то значение, которое нужно получить. В графе «Изменяя значение ячейки» — В1. Здесь должен отобразиться отобранный параметр.
  3. После нажатия ОК отобразится результат подбора. Если нужно его сохранить, вновь нажимаем ОК. В противном случае – «Отмена».

Для подбора параметра программа использует циклический процесс. Чтобы изменить число итераций и погрешность, нужно зайти в параметры Excel. На вкладке «Формулы» установить предельное количество итераций, относительную погрешность. Поставить галочку «включить итеративные вычисления».



Как решить систему уравнений матричным методом в Excel

Дана система уравнений:

  1. Значения элементов введем в ячейки Excel в виде таблицы.
  2. Найдем обратную матрицу. Выделим диапазон, куда впоследствии будут помещены элементы матрицы (ориентируемся на количество строк и столбцов в исходной матрице). Открываем список функций (fx). В категории «Математические» находим МОБР. Аргумент – массив ячеек с элементами исходной матрицы.
  3. Нажимаем ОК – в левом верхнем углу диапазона появляется значение. Последовательно жмем кнопку F2 и сочетание клавиш Ctrl + Shift + Enter.
  4. Умножим обратную матрицу Ах-1х на матрицу В (именно в таком порядке следования множителей!). Выделяем диапазон, где впоследствии появятся элементы результирующей матрицы (ориентируемся на число строк и столбцов матрицы В). Открываем диалоговое окно математической функции МУМНОЖ. Первый диапазон – обратная матрица. Второй – матрица В.
  5. Закрываем окно с аргументами функции нажатием кнопки ОК. Последовательно нажимаем кнопку F2 и комбинацию Ctrl + Shift + Enter.

Получены корни уравнений.

Решение системы уравнений методом Крамера в Excel

Возьмем систему уравнений из предыдущего примера:

Для их решения методом Крамера вычислим определители матриц, полученных заменой одного столбца в матрице А на столбец-матрицу В.

Для расчета определителей используем функцию МОПРЕД. Аргумент – диапазон с соответствующей матрицей.

Рассчитаем также определитель матрицы А (массив – диапазон матрицы А).

Определитель системы больше 0 – решение можно найти по формуле Крамера (Dx / |A|).

Для расчета Х1: =U2/$U$1, где U2 – D1. Для расчета Х2: =U3/$U$1. И т.д. Получим корни уравнений:

Решение систем уравнений методом Гаусса в Excel

Для примера возьмем простейшую систему уравнений:

3а + 2в – 5с = -1
2а – в – 3с = 13
а + 2в – с = 9

Коэффициенты запишем в матрицу А. Свободные члены – в матрицу В.

Для наглядности свободные члены выделим заливкой. Если в первой ячейке матрицы А оказался 0, нужно поменять местами строки, чтобы здесь оказалось отличное от 0 значение.

  1. Приведем все коэффициенты при а к 0. Кроме первого уравнения. Скопируем значения в первой строке двух матриц в ячейки В6:Е6. В ячейку В7 введем формулу: =B3:Е3-$B$2:$Е$2*(B3/$B$2). Выделим диапазон В7:Е7. Нажмем F2 и сочетание клавиш Ctrl + Shift + Enter. Мы отняли от второй строки первую, умноженную на отношение первых элементов второго и первого уравнения.
  2. Копируем введенную формулу на 8 и 9 строки. Так мы избавились от коэффициентов перед а. Сохранили только первое уравнение.
  3. Приведем к 0 коэффициенты перед в в третьем и четвертом уравнении. Копируем строки 6 и 7 (только значения). Переносим их ниже, в строки 10 и 11. Эти данные должны остаться неизменными. В ячейку В12 вводим формулу массива.
  4. Прямую прогонку по методу Гаусса сделали. В обратном порядке начнем прогонять с последней строки полученной матрицы. Все элементы данной строки нужно разделить на коэффициент при с. Введем в строку формулу массива: {=B12:E12/D12}.
  5. В строке 15: отнимем от второй строки третью, умноженную на коэффициент при с второй строки ({=(B11:E11-B16:E16*D11)/C11}). В строке 14: от первой строки отнимаем вторую и третью, умноженные на соответствующие коэффициенты ({=(B10:E10-B15:E15*C10-B16:E16*D10)/B10}). В последнем столбце новой матрицы получаем корни уравнения.

Примеры решения уравнений методом итераций в Excel

Вычисления в книге должны быть настроены следующим образом:

Делается это на вкладке «Формулы» в «Параметрах Excel». Найдем корень уравнения х – х3 + 1 = 0 (а = 1, b = 2) методом итерации с применением циклических ссылок. Формула:

Хn+1 = Xn– F (Xn) / M, n = 0, 1, 2, … .

M – максимальное значение производной по модулю. Чтобы найти М, произведем вычисления:

f’ (1) = -2 * f’ (2) = -11.

Полученное значение меньше 0. Поэтому функция будет с противоположным знаком: f (х) = -х + х3 – 1. М = 11.

В ячейку А3 введем значение: а = 1. Точность – три знака после запятой. Для расчета текущего значения х в соседнюю ячейку (В3) введем формулу: =ЕСЛИ(B3=0;A3;B3-(-B3+СТЕПЕНЬ(B3;3)-1/11)).

В ячейке С3 проконтролируем значение f (x): с помощью формулы =B3-СТЕПЕНЬ(B3;3)+1.

Корень уравнения – 1,179. Введем в ячейку А3 значение 2. Получим тот же результат:

Скачать решения уравнений в Excel

Корень на заданном промежутке один.

Методы решения систем уравнений — урок. Алгебра, 11 класс.

При постановке задачи — найти такие пары значений \((x;y)\), которые одновременно удовлетворяют уравнению \(p(x;y)=0\) и уравнению \(q(x;y)=0\), то говорят, что данные уравнения образуют систему уравнений:

p(x;y)=0,q(x;y)=0.

Пару значений \((x;y)\), которая одновременно является решением и первого, и второго уравнений системы, называют решением системы уравнений.

 

Решить систему уравнений — означает найти все её решения или показать, что решений нет.

Встречаются системы из трёх уравнений с тремя переменными:

p(x;y;z)=0,q(x;y;z)=0,r(x;y;z)=0.

Две системы уравнений называют равносильными, если они имеют одинаковые решения или обе системы не имеют решений.

Чтобы решить систему уравнений, можно использовать способы:

1. подстановки,

2. алгебраического сложения,

3. введения новых переменных,

4. графический.

Пример:

реши систему уравнений:

3x=y+1;7y−2x+2=7⋅7y−4x+6. y=3x−1;73x−1−2x+2=7⋅73x−1−4x+6.y=3x−1;7x+1=7⋅7−x−1+6.

 

В ходе решения подставили вместо \(y\) выражение \(3x-1\), полученное из первого уравнения.

 

Введём во втором уравнении новую переменную

t=7x+1;7−x−1=7−x+1=t−1=1t.

 

Решая второе уравнение с переменной \(t\), получим:

t=7t+6;t2−6t−7=0,t≠0;t1=−1,t2=7.

 

Возвращаясь к введённому обозначению \(t\), решаем полученные уравнения и находим \(x\):

7x+1=t↙↘7x+1=−1;7x+1=7=71;x∈∅.x+1=1;x=0.

Найдём \(y\), подставляя вместо \(x=0\).

Получим, что \(y=-1\).

Решение системы — пара чисел \((0;-1)\).

В ходе решения были использованы два метода: подстановки и введения новой переменной

Пример:

реши систему уравнений:

3x+2y=1x−y=−3|⋅2+3x+2y=12x−2y=−63x+2x+2y−2y=1+(−6);5x=−5;x=−1. x=−1;x−y=−3.x=−1;y=2.

 

Для определения решения системы использовались методы алгебраического сложения и  подстановки.

Решение систем уравнений в MathCad 13, 14, 15 на примерах. Универсальные методы Given

Для решения уравнений в Mathcad можно воспользоваться двумя способами. Эти способы были частично рассмотрены в разделе «Решение уравнений»:

Использование метода

Given — Find:

В рабочем поле mathcad записываем слово Given. Это служебное слово. Оно подключает определенные программные модули mathcad для обработки исходных данных, необходимых для решения системы уравнений численными методами.

Затем указывается начальное приближение для искомых переменных. Это нужно для увеличения скорости и точности решения системы. Если начальное приближение не задать, то mathcad по умолчанию примет его равным нулю для всех переменных, при этом, если окажется, что система имеет несколько решений, то есть риск не определить все корни. Поэтому лучше всегда задавать приближение

Рис. 1. Ввод исходных данных в поле mathcad

Далее вводятся уравнения. Их можно записать в явном или неявном виде. Само уравнение набирается с клавиатуры вручную с использованием панели Calculator. Из этой панели можно взять основные математические операции: дроби, тригонометрию, факториалы и прочее. Уравнение нужно записывать с использованием логического символа «ровно». На панели Boolean он выделен жирным шрифтом (см. рис. 2)

Рис. 2. Панели Boolean и Calculator

Когда уравнения записаны вводится функция Find(x, y, z,…) (где х, y, z,… — переменные). Это функция, которая возвращает результат решения системы. Значение функции Find() можно присвоить какой-либо переменной с помощью символа «:=» и использовать ее далее в расчетах (см. рис. 3). При решении систем уравнений в mathcad результатом всегда будет являтся матрица значений

Рис. 3. Ввод функции Find()

Для того чтобы увидеть результат решения системы уравнений, после Find(x, y, z,…) следует поставить символ «» либо «=» из панели Evaluation (см. рис. 4).

Рис. 4. Панель «Evaluation»

В зависимости от сложности системы через определенное время MathCad выведет результат. На рис. 5 можно рассмотреть синтаксис и результат решения системы уравнений. Обратите внимание, что можно присваивать результат решения системы матричной переменной и можно работать с отдельными ее элементами

Рис. 5. Результат численного решения системы уравнений

Mathcad позволяет решать системы уравний в символьном виде. Обычно это полезно, когда требуется получить не точное значение переменных, а их выражения через константы. Например, если мы заменим все числовые константы на неизвестные параметры и решим уравнение относительно x, y и z, то результат выведется в символьном виде (см. рис. 6). Причем, обратите внимание, что в данном случае нам не нужно вводить начальное приближение и мы должны использовать символ «» для вывода результата. Как правило, символьное решение получается громоздким, поэтому не всегда рекомендуется использовать этот метод

Рис. 6. Результат символьного решения системы уравнений

Использование метода

Solve:

Как показывает практика, методом solve иногда удается решить системы уравнений, которые не поддаются решению с помощью функции Find()

Синтаксис следующий: на панели matrix нажимаем иконку Matrix or Vector и в появившемся окне указываем количество уравнений входящих в систему. В нашем примере их будет три (см. рис. 7)

Рис. 7. Создание матрицы для метода SOLVE

Заполняем систему, вводя последовательно все уравнения используя логический символ «ровно» из панели Boolean. Каждый элемент матрицы-столбца содержит одно уравнение (см. рис. 8)

Рис. 8. Ввод системы уравнений для метода SOLVE

Когда все уравнения введены, убедитесь, что курсор ввода находится в вашей матрице и затем нажмите кнопку «solve» из панели Symbolic. Появится служебное слово (функция) solve. Далее поставте запятую и введите последовательно все переменные, относительно которых необходимо решить систему уравнений (см. рис. 9)

Рис. 9. Синтаксис метода SOLVE для решения систем

Уведите курсор в свободное поле mathcad и дождитесь окончания решения системы. Обратите внимание, что мы не вводили начальные приближения. Даный метод их назначает автоматически. Обратите так же внимание, что для решения системы в символьном виде синтаксис аналогичен (см. рис. 10)

Рис. 10. Синтаксис метода SOLVE для решения систем
Как показывает моя инженерная практика, решение систем в символьном виде сопряжено с большими вычислительными трудностями. То есть иногда решение системы занимает массу времени, и в итоге mathcad выдает выражение для одной переменной непомерной длины, которое нельзя использовать. 5 == 0, x]
Out[2]=

Функция Reduce сводит системы неравенств к простой форме:

(Наберите <= для ввода символа .)
In[1]:=
Reduce[{0 < x < 2, 1 <= x <= 4}, x]
Out[1]=

Упрощенная форма может состоять из нескольких интервалов:

In[2]:=
Reduce[(x - 1) (x - 2) (x - 3) (x - 4) > 0, x]
Out[2]=

Функция NumberLinePlot — это удобный способ визуализации этих результатов:

In[3]:=
NumberLinePlot[x < 1 || 2 < x < 3 || x > 4, {x, -10, 10}]
Out[3]=

Большое число уравнений и формул доступно через естественную форму ввода:

In[1]:= X
quadratic equation
Out[1]=

Справочная информация: Полиномиальные уравнения »

Справочная информация: Решение уравнений »

Hands–on Start to
Wolfram Mathematica »

Полная документация »

Demonstrations Project »

Система уравнений

Рассмотрим, что же такое система уравнений с двумя переменными, а также равносильные системы уравнений.

Например, нам даны два уравнения, которые имеют две переменные f(x;y) = 0 и g (x;y) = 0.

Решить систему уравнений – это значит найти абсолютно все общие решения двух данных уравнений, которые имеют две переменные. Каждая пара значений данных переменных, которые обращают каждое уравнение нашей системы в верное равенство и будут решением системы уравнений. Проще говоря, решить систему уравнений – это значит найти абсолютно все ее решение, или же доказать, что этих решений не существует.

Уравнения, которые образуют систему, как правило, объединяют фигурной скобкой.

Если две системы уравнений имеют одни и те же решения, то такие системы называются равносильными. Тоже касается и тех систем, которые не имеет решений. Для того, чтобы решить систему уравнений, в большинстве случаев пользуются «способом замены», то есть заменяют данную систему уравнений на более «удобную» или легкую, но, при этом. обязательно равносильную. Что касается возможности такой замены, то она обуславливается двумя теоремами:

  1. Допустим, дана система двух уравнений, которые имеют две переменные. Если оставить одно уравнений системы и не изменять его, а второе уравнение системы заменять равносильными, то мы получим систему, которая будет равносильна данной. Следствием данной теоремы является то, что, если каждое уравнение заданной системы заменять равносильными, то получим систему, которая будет равносильна данной.
  2. Допустим, дана система двух уравнений, которые имеют две переменные. Если оставить одно уравнение системы и не изменять его, а второе уравнение заменить разностью или суммой обоих уравнений системы, то полученная система уравнений будет равносильна данной.
Решение уравнения первой степени ax = b
Решение системы двух уравнений первой степени
{ ax + by = c
dx + ey = f
{ x = ce — fb (ae — db ≠ 0)
ae — db
y = af — dc
ae — db

либо через определители

x = | c  b

f  e

| ;   y = | a  c

d  f

|
| a  b

d  e

| | a  b

d  e

|
Формула корней квадратного уравнения
x1,2 = -b ± b2 — 4ac
2a
Формула приведенного квадратного уравнения
x1,2 = p
2
± 1
2
p2— 4q
Теорема Виета для квадратного уравнения
x1 + x2 = — b
a
;  x1 * x2 = c
a
Теорема Виета для приведенного квадратного уравнения
x1 + x2 = -p;  x1 * x2 = q
Теорема Виета для приведенного кубического уравнения
Разложение на множители квадратного трехчлена
ax2 + bx + c = a(x — x1)(x — x2), где x1 и x2 — корни уравнения ax2 + bx + c = 0
Выделение квадрата двучлена из квадратного трехчлена
ax2 + bx + c = a(x +b
2a
)2 +4ac — b2
  4a
Решение биквадратного уравнения
x1,2 = ± -b + √b2 — 4ac
2a
x3,4 = ± -b — √b2 — 4ac
2a
Формула действительного корня неполного кубического уравнения

Как найти решение системы уравнений

Если вы считаете, что контент, доступный с помощью Веб-сайта (как это определено в наших Условиях обслуживания), нарушает одно или более ваших авторских прав, пожалуйста, сообщите нам, предоставив письменное уведомление («Уведомление о нарушении»), содержащее в информацию, описанную ниже, назначенному агенту, указанному ниже. Если университетские наставники примут меры в ответ на ан Уведомление о нарушении, он предпримет добросовестную попытку связаться со стороной, предоставившей такой контент средства самого последнего адреса электронной почты, если таковой имеется, предоставленного такой стороной Varsity Tutors.

Ваше Уведомление о нарушении может быть направлено стороне, предоставившей контент, или третьим лицам, таким как в виде ChillingEffects.org.

Обратите внимание, что вы будете нести ответственность за ущерб (включая расходы и гонорары адвокатов), если вы существенно искажать информацию о том, что продукт или деятельность нарушают ваши авторские права. Таким образом, если вы не уверены, что содержимое находится на Веб-сайте или на который ссылается Веб-сайт, нарушает ваши авторские права, вам следует сначала обратиться к адвокату.

Чтобы подать уведомление, выполните следующие действия:

Вы должны включить следующее:

Физическая или электронная подпись владельца авторских прав или лица, уполномоченного действовать от его имени; Идентификация авторских прав, которые, как утверждается, были нарушены; Описание характера и точного местонахождения контента, который, как вы утверждаете, нарушает ваши авторские права, в \ достаточно подробно, чтобы преподаватели университета могли найти и точно идентифицировать этот контент; например, мы требуем а ссылку на конкретный вопрос (а не только название вопроса), который содержит содержание и описание к какой конкретной части вопроса — изображению, ссылке, тексту и т. д. — относится ваша жалоба; Ваше имя, адрес, номер телефона и адрес электронной почты; и Заявление от вас: (а) что вы добросовестно полагаете, что использование контента, который, как вы утверждаете, нарушает ваши авторские права не разрешены законом или владельцем авторских прав или его агентом; б) что все информация, содержащаяся в вашем Уведомлении о нарушении, является точной, и (c) под страхом наказания за лжесвидетельство вы либо владельцем авторских прав, либо лицом, уполномоченным действовать от их имени.

Отправьте жалобу нашему назначенному агенту по адресу:

Чарльз Кон Varsity Tutors LLC
101 S. Hanley Rd, Suite 300
St. Louis, MO 63105

Или заполните форму ниже:

 

5.1: Решение систем уравнений с помощью графика

Цели обучения

К концу этого раздела вы сможете:

  • Определить, является ли упорядоченная пара решением системы уравнений
  • Решите систему линейных уравнений, построив график
  • Определить количество решений линейной системы
  • Решение приложений систем уравнений с помощью графика

Примечание

Прежде чем начать, пройдите этот тест на готовность.

  1. Для уравнения \(y=\frac{2}{3}x−4\)
    ⓐ является (6,0) решением? ⓑ является (−3,−2) решением?
    Если вы пропустили эту проблему, просмотрите упражнение 2.1.1.
  2. Найдите наклон и точку пересечения с осью y линии 3x−y=12.
    Если вы пропустили эту проблему, просмотрите упражнение 4.5.7.
  3. Найдите точки пересечения x и y прямой 2x−3y=12.
    Если вы пропустили эту проблему, просмотрите упражнение 4.3.7.

Определить, является ли упорядоченная пара решением системы уравнений

В разделе «Решение линейных уравнений и неравенств» мы научились решать линейные уравнения с одной переменной.Помните, что решение уравнения — это значение переменной, которая дает истинное утверждение при подстановке в уравнение. Теперь мы будем работать с системами линейных уравнений , двумя или более линейными уравнениями, сгруппированными вместе.

Определение: СИСТЕМА ЛИНЕЙНЫХ УРАВНЕНИЙ

Когда два или более линейных уравнения группируются вместе, они образуют систему линейных уравнений.

Здесь мы сосредоточим нашу работу на системах двух линейных уравнений с двумя неизвестными.Позже вы сможете решать более крупные системы уравнений.

Ниже показан пример системы двух линейных уравнений. Мы используем фигурную скобку, чтобы показать, что два уравнения сгруппированы вместе, чтобы сформировать систему уравнений.

\[\begin{cases}{2 x+y=7} \\ {x-2 y=6}\end{cases}\]

Линейное уравнение с двумя переменными, например 2 x + y = 7, имеет бесконечное число решений. Его график представляет собой линию. Помните, что каждая точка на прямой — это решение уравнения, а каждое решение уравнения — это точка на прямой.

Чтобы решить систему двух линейных уравнений, мы хотим найти значения переменных, которые являются решениями обоих уравнений. Другими словами, мы ищем упорядоченные пары ( x , y ), которые делают оба уравнения верными. Они называются решениями системы уравнений .

ОПРЕДЕЛЕНИЕ: РЕШЕНИЕ СИСТЕМЫ УРАВНЕНИЙ

Решения системы уравнений — это значения переменных, при которых все уравнения верны.Решение системы двух линейных уравнений представлено упорядоченной парой ( x , y ).

Чтобы определить, является ли упорядоченная пара решением системы двух уравнений, мы подставляем значения переменных в каждое уравнение. Если упорядоченная пара делает оба уравнения верными, это решение системы.

Рассмотрим систему ниже:

\[\begin{case}{3x−y=7} \\ {x−2y=4}\end{cases}\]

Является ли упорядоченная пара (2,−1) решением?

Упорядоченная пара (2, −1) сделала оба уравнения верными.Поэтому (2, −1) является решением этой системы.

Попробуем другую заказанную пару. Является ли упорядоченная пара (3, 2) решением?

Упорядоченная пара (3, 2) сделала одно уравнение истинным, но другое уравнение сделало ложным. Поскольку это не решение обоих уравнений , оно не является решением этой системы.

Упражнение \(\PageIndex{1}\)

Определить, является ли упорядоченная пара решением системы: \(\begin{cases}{x−y=−1} \\ {2x−y=−5}\end{cases}\)

  1. (-2,-1)
  2. (-4,-3)
Ответить

1.

(−2, −1) не делает оба уравнения верными. (−2, −1) не является решением.

2.

(−4, −3) не делает оба уравнения верными. (−4, −3) является решением.

Упражнение \(\PageIndex{2}\)

Определить, является ли упорядоченная пара решением системы: \(\begin{cases}{3x+y=0} \\ {x+2y=−5}\end{cases}\)

  1. (1,−3)
  2. (0,0)
Ответить
  1. да
  2. нет

Упражнение \(\PageIndex{3}\)

Определить, является ли упорядоченная пара решением системы: \(\begin{cases}{x−3y=−8} \\ {−3x−y=4}\end{cases}\)

  1. (2,−2)
  2. (−2,2)
Ответить
  1. нет
  2. да

Решение системы линейных уравнений с помощью графика

В этой главе мы будем использовать три метода для решения системы линейных уравнений. Первый метод, который мы будем использовать, — это построение графика. График линейного уравнения представляет собой прямую. Каждая точка на прямой является решением уравнения. Для системы двух уравнений мы начертим две линии. Тогда мы сможем увидеть все точки, являющиеся решениями каждого уравнения. И, найдя, что общего у линий, мы найдем решение системы.

Большинство линейных уравнений с одной переменной имеют одно решение, но мы видели, что некоторые уравнения, называемые противоречиями , не имеют решений, а для других уравнений, называемых тождествами, решениями являются все числа.Точно так же, когда мы решаем систему двух линейных уравнений, представленную графиком из двух линий на одной плоскости, возможны три случая, как показано на рисунке \(\PageIndex{1}\):

Рисунок \(\PageIndex{1}\)

В качестве первого примера решения системы линейных уравнений в этом разделе и в следующих двух разделах мы будем решать одну и ту же систему из двух линейных уравнений. Но мы будем использовать другой метод в каждом разделе. Увидев третий метод, вы решите, какой метод был наиболее удобным для решения этой системы.

Упражнение \(\PageIndex{4}\): Как решить систему линейных уравнений с помощью графика

Решите систему, построив график: \(\begin{cases}{2x+y=7} \\ {x−2y=6}\end{cases}\)

Ответить

Упражнение \(\PageIndex{5}\)

Решите каждую систему, построив график: \(\begin{cases}{x−3y=−3} \\ {x+y=5}\end{cases}\)

Ответить

(3,2)

Упражнение \(\PageIndex{6}\)

Решите каждую систему, построив график: \(\begin{cases}{−x+y=1} \\ {3x+2y=12}\end{cases}\)

Ответить

(2,3)

Шаги, необходимые для решения системы линейных уравнений с помощью графика, показаны ниже.

РЕШИТЬ СИСТЕМУ ЛИНЕЙНЫХ УРАВНЕНИЙ С ПОМОЩЬЮ ГРАФИКА.

  1. Нарисуйте первое уравнение.
  2. Постройте график второго уравнения в той же прямоугольной системе координат.
  3. Определите, пересекаются ли линии, параллельны или являются одной и той же линией.
  4. Определите решение системы.
    • Если линии пересекаются, определите точку пересечения. Убедитесь, что это решение обоих уравнений. Это решение системы.
    • Если прямые параллельны, система не имеет решения.
    • Если линии одинаковые, система имеет бесконечное число решений.

Упражнение \(\PageIndex{7}\)

Решите систему, построив график: \(\begin{cases}{y=2x+1} \\ {y=4x−1}\end{cases}\)

Ответить

Оба уравнения в этой системе представлены в форме наклона и пересечения, поэтому мы будем использовать их наклоны и y -пересечения для их построения. \(\begin{cases}{y=2x+1} \\ {y=4x−1}\end{cases}\)

” «>
Найдите наклон и y -пересечение первого уравнения
.
Найдите наклон и y -пересечение первого уравнения
.
Нарисуйте две линии.
Определите точку пересечения. Линии пересекаются в (1, 3).
Проверьте решение обоих уравнений. \(\begin{array}{l}{y=2 x+1} & {y = 4x — 1}\\{3\stackrel{?}{=}2 \cdot 1+1} &{3\ stackrel{?}{=}4 \cdot 1-1} \\ {3=3 \checkmark}&{3=3 \checkmark} \end{массив}\)
Решение (1, 3).

Упражнение \(\PageIndex{8}\)

Решите систему графически: \(\begin{cases}{y=2x+2} \\ {y=-x−4}\end{cases}\)

Ответить

(-2,-2)

    Упражнение \(\PageIndex{9}\)

    Решите систему, построив график: \(\begin{cases}{y=3x+3} \\ {y=-x+7}\end{cases}\)

    Ответить

    (1,6)

    Оба уравнения в упражнении \(\PageIndex{7}\) были даны в форме наклона и пересечения.Это упростило нам быстрое графическое построение линий. В следующем примере мы сначала перепишем уравнения в форме наклона и пересечения.

    Упражнение \(\PageIndex{10}\)

    Решите систему с помощью графика: \(\begin{cases}{3x+y=−1} \\ {2x+y=0}\end{cases}\)

    Ответить

    Мы решим оба этих уравнения относительно yy, чтобы мы могли легко построить их график, используя их наклоны и y -перехваты. \(\begin{cases}{3x+y=−1} \\ {2x+y=0}\end{cases}\)

    ” The first equation shows 3x + y = -1. Then 3(-1) + 2 = -1. And then -1 = -1. The second equation shows 2x + y = 0. Then 2(-1) + 2 = 0. Then 0 = 0. The figure then says, “The solution is (-1, 2).”»>
    Решите первое уравнение для y .

    Найти наклон и у -пересечение.

    Решите второе уравнение для y .

    Найти наклон и y -пересечение.

    \(\begin{align} 3 x+y &=-1 \\ y &=-3 x-1 \\ m &=-3 \\ b &=-1 \\ 2 x+y &=0 \ \ y &=-2 x \\ b &=0 \end{выровнено}\)
    Нарисуйте линии.
    Определите точку пересечения. Линии пересекаются в (−1, 2).
    Проверьте решение обоих уравнений. \(\begin{array}{rllrll}{3x+y}&{=}&{-1} & {2x +y}&{=}&{0}\\{3(-1)+ 2} &{\stackrel{?}{=}}&{-1} &{2(-1)+2}&{\stackrel{?}{=}}&{0} \\ {-1}&{= }&{-1 \checkmark}&{0}&{=}&{0 \checkmark} \end{массив}\)
    Решение (−1, 2).

    Упражнение \(\PageIndex{11}\)

    Решите каждую систему, построив график: \(\begin{cases}{−x+y=1} \\ {2x+y=10}\end{cases}\)

    Ответить

    (3,4)

    Упражнение \(\PageIndex{12}\)

    Решите каждую систему, построив график: \(\begin{cases}{ 2x+y=6} \\ {x+y=1}\end{cases}\)

    Ответить

    (5,−4)

    Обычно, когда уравнения задаются в стандартной форме, наиболее удобным способом их графического отображения является использование точек пересечения. Мы сделаем это в упражнении \(\PageIndex{13}\).

    Упражнение \(\PageIndex{13}\)

    Решите систему, построив график: \(\begin{cases}{x+y=2} \\ {x−y=4}\end{cases}\)

    Ответить

    Мы найдем точки пересечения x и y обоих уравнений и используем их для построения линий.

    Упражнение \(\PageIndex{14}\)

    Решите каждую систему, построив график: \(\begin{cases}{x+y=6} \\ {x−y=2}\end{cases}\)

    Ответить

    (4,2)

    Упражнение \(\PageIndex{15}\)

    Решите каждую систему, построив график: \(\begin{cases}{x+y=2} \\ {x−y=-8}\end{cases}\)

    Ответить

    (5,−3)

    Вы помните, как построить линейное уравнение с одной переменной? Это будет либо вертикальная, либо горизонтальная линия.

    Упражнение \(\PageIndex{16}\)

    Решите систему, построив график: \(\begin{cases}{y=6} \\ {2x+3y=12}\end{cases}\)

    Ответить

    Упражнение \(\PageIndex{17}\)

    Решите каждую систему, построив график: \(\begin{cases}{y=−1} \\ {x+3y=6}\end{cases}\)

    Ответить

    (9,−1)

    Упражнение \(\PageIndex{18}\)

    Решите каждую систему, построив график: \(\begin{cases}{x=4} \\ {3x−2y=24}\end{cases}\)

    Ответить

    (4,−6)

    Во всех системах линейных уравнений линии пересекались, а решением была одна точка.В следующих двух примерах мы рассмотрим систему уравнений, не имеющую решения, и систему уравнений, имеющую бесконечное число решений.

    Упражнение \(\PageIndex{19}\)

    Решите систему с помощью графика: \(\begin{cases}{y=\frac{1}{2}x−3} \\ {x−2y=4}\end{cases}\)

    Ответить

    Упражнение \(\PageIndex{20}\)

    Решите каждую систему, построив график: \(\begin{cases}{y=-\frac{1}{4}x+2} \\ {x+4y=-8}\end{cases}\)

    Ответить

    нет решения

    Упражнение \(\PageIndex{21}\)

    Решите каждую систему с помощью графика: \(\begin{cases}{y=3x−1} \\ {6x−2y=6}\end{cases}\)

    Ответить

    нет решения

    Упражнение \(\PageIndex{22}\)

    Решите систему, построив график: \(\begin{cases}{y=2x−3} \\ {−6x+3y=-9}\end{cases}\)

    Ответить

    Упражнение \(\PageIndex{23}\)

    Решите каждую систему, построив график: \(\begin{cases}{y=−3x−6} \\ {6x+2y=-12}\end{cases}\)

    Ответить

    бесконечно много решений

    Упражнение \(\PageIndex{24}\)

    Решите каждую систему с помощью графика: \(\begin{cases}{y=\frac{1}{2}x−4} \\ {2x−4y=16}\end{cases}\)

    Ответить

    бесконечно много решений

    Если вы запишете второе уравнение в упражнении \(\PageIndex{22}\) в форме наклона и пересечения, вы увидите, что уравнения имеют один и тот же наклон и одинаковые y -отсечение.

    Когда мы рисовали вторую линию в последнем примере, мы нарисовали ее прямо над первой линией. Мы говорим, что две прямые совпадают. Совпадающие линии имеют одинаковый наклон и одинаковую точку пересечения и .

    СОВПАДАЮЩИЕ ЛИНИИ

    Совпадающие линии имеют одинаковый наклон и одинаковую точку пересечения и .

    Определить количество решений линейной системы

    Будут времена, когда мы захотим узнать, сколько решений будет у системы линейных уравнений, но на самом деле нам может не понадобиться находить решение.Будет полезно определить это без графика.

    Мы видели, что две прямые в одной плоскости должны либо пересекаться, либо быть параллельными. Все системы уравнений от Упражнения \(\PageIndex{4}\) до Упражнения \(\PageIndex{16}\) состояли из двух пересекающихся линий. Каждая система имела одно решение.

    Система с параллельными линиями, такая как Упражнение \(\PageIndex{19}\), не имеет решения. Что произошло в упражнении \(\PageIndex{22}\)? У уравнений совпадающих прямых , поэтому система имеет бесконечно много решений.

    Мы упорядочим эти результаты на рисунке \(\PageIndex{2}\) ниже:

    Рисунок \(\PageIndex{2}\)

    Параллельные линии имеют одинаковый наклон, но разные и точки пересечения. Итак, если мы запишем оба уравнения в виде системы линейных уравнений в форме наклон-пересечение, мы увидим, сколько будет решений без построения графика! Посмотрите на систему, которую мы решили в упражнении \(\PageIndex{19}\).

    \(\begin{array} {cc} & \begin{cases}{y=\frac{1}{2}x−3} \\ {x−2y=4}\end{cases}\\ \text {Первая строка находится в форме пересечения наклона.} &\text { Если решить второе уравнение относительно } y, \text { получим } \\ &x-2 y =4 \\ y = \frac{1}{2}x -3& x-2 y =- x+4 \\ &y =\frac{1}{2} x-2 \\ m=\frac{1}{2}, b=-3&m=\frac{1}{2}, b=-2 \ конец{массив}\)

    Две линии имеют одинаковый наклон, но разные и -пересечения. Это параллельные линии.

    На рисунке \(\PageIndex{3}\) показано, как определить количество решений линейной системы, глядя на наклоны и точки пересечения.

    Рисунок \(\PageIndex{3}\)

    Давайте еще раз взглянем на наши уравнения в упражнении \(\PageIndex{19}\), которые дали нам параллельные линии.

    \[\begin{cases}{y=\frac{1}{2}x−3} \\ {x−2y=4}\end{cases}\)]

    Когда обе линии были в форме пересечения наклона, мы имели:

    \[y=\frac{1}{2} x-3 \quad y=\frac{1}{2} x-2\]

    Вы понимаете, что невозможно иметь одну упорядоченную пару (x,y), являющуюся решением обоих этих уравнений?

    Мы называем такую ​​систему уравнений противоречивой системой . У него нет решения.

    Система уравнений, которая имеет хотя бы одно решение, называется согласованной системой .

    ПОСЛЕДОВАТЕЛЬНЫЕ И НЕСООТВЕТСТВУЮЩИЕ СИСТЕМЫ

    непротиворечивая система уравнений — это система уравнений, имеющая хотя бы одно решение.

    несовместимая система уравнений — это система уравнений, не имеющая решения.

    Мы также классифицируем уравнения в системе уравнений, называя уравнения независимыми или зависимыми . Если два уравнения представляют собой независимых уравнений , каждое из них имеет собственный набор решений.Пересекающиеся прямые и параллельные прямые независимы.

    Если два уравнения являются зависимыми, то все решения одного уравнения также являются решениями другого уравнения. Когда мы рисуем два зависимых уравнения , мы получаем совпадающие линии.

    НЕЗАВИСИМЫЕ И ЗАВИСИМЫЕ УРАВНЕНИЯ

    Два уравнения являются независимыми , если они имеют разные решения.

    Два уравнения являются зависимыми , если все решения одного уравнения являются также решениями другого уравнения.

    Подведем итоги, взглянув на графики трех типов систем. См. рисунок \(\PageIndex{4}\) и рисунок \(\PageIndex{5}\).

    Рисунок \(\PageIndex{4}\)Рисунок \(\PageIndex{5}\)

    Упражнение \(\PageIndex{25}\)

    Без построения графика определите количество решений, а затем классифицируйте систему уравнений: \(\begin{cases}{y=3x−1} \\ {6x−2y=12}\end{cases}\)

    Ответить

    \(\begin{array}{lrrl} \text{Мы будем сравнивать наклоны и пересечения} & \begin{cases}{y=3x−1} \\ {6x−2y=12}\end{cases} \ \ \text{из двух строк.} \\ \text{Первое уравнение уже введено} \\ \text{форма пересечения наклона.} \\ & {y = 3x — 1}\\ \text{Запишите второе уравнение} \\ \text{ наклон-пересечение.} \\ & 6x-2y &=&12 \\ & -2y &=& -6x — 12 \\ &\frac{-2y}{-2} &=& \frac{-6x + 12 }{-2}\\ &y&=&3x-6\\\\ \text{Найдите наклон и точку пересечения каждой прямой.} & y = 3x-1 & y=3x-6 \\ &m = 3 & m = 3 \\&b=-1 &b=-6 \\ \text{Поскольку наклоны одинаковы, а точки пересечения} \\ \text{различны, прямые параллельны.}\конец{массив}\)

    Система уравнений, графики которой представляют собой параллельные прямые, не имеет решения, является противоречивой и независимой.

    Упражнение \(\PageIndex{26}\) ​​

    Без построения графика определите количество решений, а затем классифицируйте систему уравнений.

    \(\begin{case}{y=−2x−4} \\ {4x+2y=9}\end{case}\)

    Ответить

    нет решения, противоречиво, независимо

    Упражнение \(\PageIndex{27}\)

    Без построения графика определите количество решений, а затем классифицируйте систему уравнений.

    \(\begin{case}{y=\frac{1}{3}x−5} \\ {x-3y=6}\end{cases}\)

    Ответить

    нет решения, противоречиво, независимо

    Упражнение \(\PageIndex{28}\)

    Без построения графика определите количество решений, а затем классифицируйте систему уравнений: \(\begin{cases}{2x+y=−3} \\ {x−5y=5}\end{cases}\)

    Ответить

    \(\begin{array}{lrrlrl} \text{Мы будем сравнивать наклоны и пересечения} & \begin{cases}{2x+y=-3} \\ {x−5y=5}\end{cases} \\ \text{из двух строк. } \\ \text{Второе уравнение запишите} \\ \text{форма наклона–отрезка.} \\ &2x+y&=&-3 & x−5y&=&5\\ & y &=& -2x -3 & -5y &=&-x+5 \\ &&&&\frac{-5y}{-5} &=& \frac{-x + 5}{-5}\\ &&&&y&=&\frac{1}{5} x-1\\\\ \text{Найдите наклон и точку пересечения каждой линии.} & y &=& -2x-3 & y&=&\frac{1}{5}x-1 \\ &m &=& -2 & m &=& \frac{1}{5} \\&b&=&-3 &b&=&-1 \\ \text{Поскольку наклоны одинаковы, а y-перехваты} \\ \text{различны, линии параллельны.}\конец{массив}\)

    Система уравнений, графики которой пересекаются, имеет 1 решение, непротиворечива и независима.

    Упражнение \(\PageIndex{29}\)

    Без построения графика определите количество решений, а затем классифицируйте систему уравнений.

    \(\begin{case}{3x+2y=2} \\ {2x+y=1}\end{case}\)

    Ответить

    одно решение, согласованное, независимое

    Упражнение \(\PageIndex{30}\)

    Без построения графика определите количество решений, а затем классифицируйте систему уравнений.

    \(\begin{case}{x+4y=12} \\ {−x+y=3}\end{case}\)

    Ответить

    одно решение, согласованное, независимое

    Упражнение \(\PageIndex{31}\)

    Без построения графика определите количество решений, а затем классифицируйте систему уравнений. \(\begin{cases}{3x−2y=4} \\ {y=\frac{3}{2}x−2}\end{cases}\)

    Ответить

    \(\begin{array}{lrrlrl} \text{Мы сравним наклоны и пересечения двух прямых.}& \begin{case}{3x−2y} &=&{4} \\ {y}&=&{\frac{3}{2}x−2}\end{cases} \\ \text{Write второе уравнение в} \\ \text{наклон – точка пересечения.} \\ &3x-2y&=&4 \\ & -2y &=& -3x +4 \\ &\frac{-2y}{-2} &= & \frac{-3x + 4}{-2}\\ &y&=&\frac{3}{2}x-2\\\\ \text{Найдите наклон и точку пересечения каждой линии.} &y&=&\ frac{3}{2}x-2\\ \text{Поскольку уравнения одинаковы, они имеют одинаковый наклон} \\ \text{и одинаковый перехват, поэтому линии совпадают.}\конец{массив}\)

    Система уравнений, графики которой представляют собой совпадающие прямые, имеет бесконечно много решений, непротиворечива и зависима.

    Упражнение \(\PageIndex{32}\)

    Без построения графика определите количество решений, а затем классифицируйте систему уравнений.

    \(\begin{case}{4x−5y=20} \\ {y=\frac{4}{5}x−4}\end{cases}\)

    Ответить

    бесконечно много решений, непротиворечивых, зависимых

    Упражнение \(\PageIndex{33}\)

    Без построения графика определите количество решений, а затем классифицируйте систему уравнений.

    \(\begin{case}{−2x−4y=8} \\ {y=−\frac{1}{2}x−2}\end{cases}\)

    Ответить

    бесконечно много решений, непротиворечивых, зависимых

    Решение приложений систем уравнений с помощью графика

    Мы будем использовать ту же стратегию решения задач, что и в Математические модели , для настройки и решения приложений систем линейных уравнений. Здесь мы немного изменим стратегию, чтобы она подходила для систем уравнений.

    ИСПОЛЬЗУЙТЕ СТРАТЕГИЮ РЕШЕНИЯ ЗАДАЧ ДЛЯ СИСТЕМ ЛИНЕЙНЫХ УРАВНЕНИЙ.

    1. Прочитайте проблему. Убедитесь, что все слова и идеи понятны.
    2. Определите , что мы ищем.
    3. Имя то что мы ищем. Выберите переменные для представления этих величин.
    4. Переведите в систему уравнений.
    5. Решите систему уравнений, используя хорошие методы алгебры.
    6. Проверьте ответ в задаче и убедитесь, что он имеет смысл.
    7. Ответьте на вопрос полным предложением.

    На шаге 5 мы будем использовать метод, представленный в этом разделе. Составим графики уравнений и найдем решение.

    Упражнение \(\PageIndex{34}\)

    Сондра готовит 10 литров пунша из фруктового сока и содовой. Количество литров фруктового сока в 4 раза превышает количество литров газировки. Сколько литров фруктового сока и сколько литров газированной воды нужно Сондре?

    Ответить

    Шаг 1. Прочтите проблему.

    Шаг 2. Определите , что мы ищем.

    Мы ищем количество литров фруктового сока и количество литров газированной воды, которые понадобятся Сондре.

    Шаг 3. Назовите то, что мы ищем. Выберите переменные для представления этих величин.

      Пусть f= количество литров фруктового сока.
        c= количество литров содовой

    Шаг 4. Переведите в систему уравнений.

    Теперь у нас есть система. \(\begin{cases}{f+c=10} \\ {f=4c}\end{cases}\)

    Шаг 5. Решите систему уравнений, используя хорошие методы алгебры.

    Точка пересечения (2, 8) является решением. Это означает, что Сондре нужно 2 литра газированной воды и 8 литров фруктового сока.

    Шаг 6. Проверьте ответ в задаче и убедитесь, что он имеет смысл.

    Имеет ли это смысл в задаче?

    Да, количество литров фруктового сока, 8, в 4 раза превышает количество литров газированной воды, 2.

    Да, 10 литров пунша — это 8 литров фруктового сока плюс 2 литра содовой.

    Шаг 7. Ответьте на вопрос полным предложением.

    Сондре нужно 8 литров фруктового сока и 2 литра содовой.

    Упражнение \(\PageIndex{35}\)

    Мэнни готовит 12 литров апельсинового сока из концентрата и воды. Количество литров воды в 3 раза превышает количество литров концентрата. Сколько литров концентрата и сколько литров воды нужно Мэнни?

    Ответить

    Мэнни нужно 3 литра концентрата сока и 9 литров воды.

    Упражнение \(\PageIndex{36}\)

    Алиша готовит кофейный напиток на 18 унций из заваренного кофе и молока. Количество унций сваренного кофе в 5 раз превышает количество унций молока. Сколько унций кофе и сколько унций молока нужно Алише?

    Ответить

    Алише нужно 15 унций кофе и 3 унции молока.

    Примечание

    Доступ к этим онлайн-ресурсам для получения дополнительных инструкций и практики решения систем уравнений с помощью графиков.

    Ключевые понятия

    • Решение системы линейных уравнений с помощью графика
      1. Нарисуйте первое уравнение.
      2. Постройте график второго уравнения в той же прямоугольной системе координат.
      3. Определите, пересекаются ли линии, параллельны или являются одной и той же линией.
      4. Определите решение системы.
        Если линии пересекаются, определите точку пересечения. Убедитесь, что это решение обоих уравнений. Это решение системы.
        Если прямые параллельны, система не имеет решения.
        Если линии одинаковые, система имеет бесконечное число решений.
      5. Проверьте решение обоих уравнений.
    • Определить количество решений по графу линейной системы

    • Определите количество решений линейной системы, глядя на наклоны и точки пересечения

    • Определить количество решений и классифицировать систему уравнений

    • Стратегия решения задач для систем линейных уравнений
      1. Прочитайте проблему.Убедитесь, что все слова и идеи понятны.
      2. Определите , что мы ищем.
      3. Имя то что мы ищем. Выберите переменные для представления этих величин.
      4. Переведите в систему уравнений.
      5. Решите систему уравнений, используя хорошие методы алгебры.
      6. Проверьте ответ в задаче и убедитесь, что он имеет смысл.
      7. Ответьте на вопрос полным предложением.

    Глоссарий

    совпадающие строки
    Совпадающие линии — это линии, имеющие одинаковый наклон и одинаковую точку пересечения и .
    последовательная система
    Непротиворечивая система уравнений — это система уравнений, имеющая хотя бы одно решение.
    зависимые уравнения
    Два уравнения являются зависимыми, если все решения одного уравнения являются также решениями другого уравнения.
    несовместимая система
    Несовместная система уравнений — это система уравнений, не имеющая решения.
    независимые уравнения
    Два уравнения независимы, если они имеют разные решения.
    решения системы уравнений
    Решениями системы уравнений являются значения переменных, при которых все уравнения верны.Решение системы двух линейных уравнений представлено упорядоченной парой ( x , y ).
    система линейных уравнений
    Когда два или более линейных уравнения группируются вместе, они образуют систему линейных уравнений.

    5.3: Решение систем уравнений методом исключения

    Навыки для развития

    К концу этого раздела вы сможете:

    • Решить систему уравнений методом исключения
    • Решение приложений систем уравнений методом исключения
    • Выберите наиболее удобный способ решения системы линейных уравнений

    Примечание

    Прежде чем начать, пройдите этот тест на готовность.

    1. Упрощение −5(6−3a).
      Если вы пропустили эту проблему, просмотрите [ссылка] .
    2. Решите уравнение \(\frac{1}{3}x+\frac{5}{8}=\frac{31}{24}\).
      Если вы пропустили эту проблему, просмотрите [ссылка] .

    Мы решили системы линейных уравнений с помощью графиков и подстановок. Графики хорошо работают, когда переменные коэффициенты малы, а решение имеет целые значения. Подстановка работает хорошо, когда мы можем легко решить одно уравнение для одной из переменных и не иметь слишком много дробей в результирующем выражении.

    Третий метод решения систем линейных уравнений называется методом исключения. Когда мы решали систему подстановкой, мы начинали с двух уравнений и двух переменных и сводили ее к одному уравнению с одной переменной. То же самое мы сделаем и с методом исключения, но у нас будет другой способ добраться туда.

    Метод исключения основан на свойстве сложения равенства. Свойство сложения равенства говорит о том, что, когда вы добавляете одно и то же количество к обеим частям уравнения, вы все равно получаете равенство. Мы расширим свойство равенства сложения, чтобы сказать, что когда вы добавляете равные количества к обеим частям уравнения, результаты равны.

    Для любых выражений a , b , c и d ,

    \[\begin{array}{lc} \text{ if } & a=b \\ \text { and } & c=d \\ \text { then } &a+c =b+d \end{array} \]

    Чтобы решить систему уравнений методом исключения, мы начинаем с обоих уравнений в стандартной форме. Затем мы решаем, какую переменную будет проще всего исключить.Как мы решаем? Мы хотим, чтобы коэффициенты одной переменной были противоположными, чтобы мы могли сложить уравнения и исключить эту переменную.

    Обратите внимание, как это работает, когда мы складываем эти два уравнения вместе:

    \[\begin{массив}{l} 3x+y=5 \\ \underline{2x-y=0} \\ 5x\quad\quad=5\end{массив}\]

    и прибавляются к нулю, и у нас есть одно уравнение с одной переменной.

    Давайте попробуем еще:

    \[\left\{\begin{массив}}{l}{x+4 y=2} \\ {2x+5 y=-2}\end{массив}\right.\]

    На этот раз мы не видим переменную, которую можно сразу исключить, если добавить уравнения.

    Но если мы умножим первое уравнение на -2, мы сделаем коэффициенты x противоположными. Мы должны умножить каждый член в обеих частях уравнения на −2.

    Теперь мы видим, что коэффициенты членов x являются противоположными, поэтому x будут исключены, когда мы добавим эти два уравнения.

    Сложите уравнения самостоятельно — результат должен быть −3 y = −6.И это кажется легко решить, не так ли? Вот как это будет выглядеть.

    Сделаем еще:

    \[\left\{\begin{array}{l}{4 x-3 y=10} \\ {3 x+5 y=-7}\end{array}\right.\]

    Не похоже, что мы можем сделать коэффициенты одной переменной противоположными, умножив одно из уравнений на константу, если только мы не используем дроби. Поэтому вместо этого нам придется умножить оба уравнения на константу.

    Мы можем сделать коэффициенты x противоположными, если умножим первое уравнение на 3, а второе на -4, так что мы получим 12 x и -12 x .

    Это дает нам два новых уравнения:

    \[\left\{\begin{выровнено} 12 x-9 y &=30 \\-12 x-20 y &=28 \end{выровнено}\right.\]

    Когда мы сложим эти уравнения,

    \[\[\left\{\begin{array}{r}{12 x-9 y=30} \\ {\underline{-12 x-20 y=28}} \\\end{array}\ вправо.\\\quad\qquad {-29 y=58}\]\]

    x исключены, и мы просто имеем −29 y = 58.

    Как только мы получим уравнение с одной переменной, мы решим его.Затем мы подставляем это значение в одно из исходных уравнений, чтобы найти оставшуюся переменную. И, как всегда, мы проверяем наш ответ, чтобы убедиться, что он является решением обоих исходных уравнений.

    Теперь мы посмотрим, как использовать исключение для решения той же системы уравнений, которую мы решали с помощью графика и подстановки.

    Упражнение \(\PageIndex{2}\)

    Решите систему методом исключения. \(\left\{\begin{массив}}{l}{3 x+y=5} \\ {2 x-3 y=7}\end{массив}\right.\)

    Ответить

    (2,−1)

    Упражнение \(\PageIndex{3}\)

    Решите систему методом исключения.\(\left\{\begin{массив}}{l}{4 x+y=-5} \\ {-2 x-2 y=-2}\end{массив}\right.\)

    Ответить

    (−2,3)

    Шаги перечислены ниже для удобства.

    КАК РЕШИТЬ СИСТЕМУ УРАВНЕНИЙ ИСКЛЮЧЕНИЕМ.

    1. Запишите оба уравнения в стандартной форме. Если какие-либо коэффициенты являются дробями, очистите их.
    2. Сделайте коэффициенты одной переменной противоположными.
      • Решите, какую переменную вы удалите.
      • Умножьте одно или оба уравнения так, чтобы коэффициенты этой переменной были противоположны.
    3. Добавьте уравнения, полученные на шаге 2, чтобы исключить одну переменную.
    4. Найдите оставшуюся переменную.
    5. Подставьте решение шага 4 в одно из исходных уравнений. Затем найдите другую переменную.
    6. Запишите решение в виде упорядоченной пары.
    7. Убедитесь, что упорядоченная пара является решением обоих исходных уравнений.

    Сначала мы сделаем пример, в котором мы можем сразу исключить одну переменную.

    Упражнение \(\PageIndex{4}\)

    Решите систему методом исключения. \(\left\{\begin{массив}}{l}{x+y=10} \\ {xy=12}\end{массив}\right.\)

    Ответить

    Упражнение \(\PageIndex{5}\)

    Решите систему методом исключения. \(\left\{\begin{массив}}{l}{2 x+y=5} \\ {xy=4}\end{массив}\right. \)

    Ответить

    (3,−1)

    Упражнение \(\PageIndex{6}\)

    Решите систему методом исключения.\(\left\{\begin{массив}}{l}{x+y=3} \\ {-2 xy=-1}\end{массив}\right.\)

    Ответить

    (−2,5)

    В упражнении \(\PageIndex{7}\) мы сможем сделать коэффициенты одной переменной противоположными, умножив одно уравнение на константу.

    Упражнение \(\PageIndex{7}\)

    Решите систему методом исключения. \(\left\{\begin{array}{l}{3 x-2 y=-2} \\ {5 x-6 y=10}\end{array}\right.\)

    Ответить

    Упражнение \(\PageIndex{8}\)

    Решите систему методом исключения.\(\left\{\begin{массив}}{l}{4 x-3 y=1} \\ {5 x-9 y=-4}\end{массив}\right.\)

    Ответить

    (1,1)

    Упражнение \(\PageIndex{9}\)

    Решите систему методом исключения. \(\left\{\begin{array}{l}{3 x+2 y=2} \\ {6 x+5 y=8}\end{array}\right.\ )

    Ответить

    (−2,4)

    Теперь сделаем пример, где нам нужно умножить оба уравнения на константы, чтобы сделать коэффициенты одной переменной противоположными.

    Упражнение \(\PageIndex{10}\)

    Решите систему методом исключения. \(\left\{\begin{array}{l}{4 x-3 y=9} \\ {7 x+2 y=-6}\end{array}\right.\)

    Ответить

    В этом примере мы не можем умножить только одно уравнение на любую константу, чтобы получить противоположные коэффициенты. Поэтому мы стратегически умножим оба уравнения на константу, чтобы получить противоположности.

    Упражнение \(\PageIndex{11}\)

    Решите систему методом исключения.\(\left\{\begin{array}{l}{3 x-4 y=-9} \\ {5 x+3 y=14}\end{array}\right.\)

    Ответить

    (1,3)

    Упражнение \(\PageIndex{12}\)

    Решите систему методом исключения. \(\left\{\begin{массив}}{l}{7 x+8 y=4} \\ {3 x-5 y=27}\end{массив}\right.\)

    Ответить

    (4,−3)

    Когда система уравнений содержит дроби, мы сначала очистим дроби, умножив каждое уравнение на его ЖК-дисплей.

    Упражнение \(\PageIndex{13}\)

    Решите систему методом исключения. \(\left\{\begin{array}{l}{x+\frac{1}{2} y=6} \\ {\frac{3}{2} x+\frac{2}{3} y= \frac{17}{2}}\end{массив}\right.\)

    Ответить

    В этом примере оба уравнения содержат дроби. Нашим первым шагом будет умножение каждого уравнения на его ЖК-дисплей, чтобы очистить дроби.

    Упражнение \(\PageIndex{14}\)

    Решите систему методом исключения.\(\left\{\begin{array}{l}{\frac{1}{3} x-\frac{1}{2} y=1} \\ {\frac{3}{4} xy= \frac{5}{2}}\end{массив}\right.\)

    Ответить

    (6,2)

    Упражнение \(\PageIndex{15}\)

    Решите систему методом исключения. \(\left\{\begin{array}{l}{x+\frac{3}{5} y=-\frac{1}{5}} \\ {-\frac{1}{2} x- \frac{2}{3} y=\frac{5}{6}}\end{массив}\right.\)

    Ответить

    (1,−2)

    В Решении систем уравнений с помощью графика мы видели, что не все системы линейных уравнений имеют одну упорядоченную пару в качестве решения.Когда два уравнения действительно представляли собой одну прямую, решений было бесконечно много. Мы назвали это последовательной системой. Когда два уравнения описывали параллельные прямые, решения не было. Мы назвали это непоследовательной системой.

    Упражнение \(\PageIndex{16}\)

    Решите систему методом исключения.\(\left\{\begin{array}{l}{3 x+4 y=12} \\ {y=3-\frac{3}{4} x}\end{ массив}\справа.\)

    Ответить

    \(\begin{array} {ll} & \left\{\begin{align} 3 x+4 y &=12 \\ y &=3-\frac{3}{4} x \end{align} \правильно.\\\\\text{Запишите второе уравнение в стандартной форме. } & \left\{\begin{array}{l}{3 x+4 y=12} \\ {\frac{3}{4} x +y=3}\end{array}\right.\\ \\ \text{Очистите дроби, умножив второе уравнение на 4.} & \left\{\begin{align} 3 x+4 y &=12 \ \ 4\left(\frac{3}{4} x+y\right) &=4(3) \end{выровнено}\right. \\\\ \text{Упростить.} & \left\{\begin{array}{l}{3 x+4 y=12} \\ {3 x+4 y=12}\end{array}\right .\\\\ \text{Чтобы исключить переменную, мы умножаем второе уравнение на −1.} & \left\{\begin{array}{c}{3 x+4 y=12} \\ \underline{- 3 x-4 y=-12} \end{массив}\right.\\ &\qquad\qquad\quad 0=0 \\ \text{Упростить и добавить.} \end{массив}\)

    Это верное утверждение. Уравнения непротиворечивы, но зависимы. Их графики были бы одной линией. Система имеет бесконечно много решений.

    После того, как мы очистили дроби во втором уравнении, вы заметили, что два уравнения были одинаковыми? Это означает, что у нас есть совпадающие линии.

    Упражнение \(\PageIndex{17}\)

    Решите систему методом исключения.\(\left\{\begin{array}{l}{5 x-3 y=15} \\ {y=-5+\frac{5}{3} x}\end{array}\right. \ )

    Ответить

    бесконечно много решений

    Упражнение \(\PageIndex{18}\)

    Решите систему методом исключения. \(\left\{\begin{array}{l}{x+2 y=6} \\ {y=-\frac{1}{2} x+3}\end{array}\right.\)

    Ответить

    бесконечно много решений

    Упражнение \(\PageIndex{19}\)

    Решите систему методом исключения.\(\left\{\begin{массив}}{l}{-6 x+15 y=10} \\ {2 x-5 y=-5}\end{массив}\right.\)

    Ответить

    \(\begin{array} {ll} \text{Уравнения в стандартной форме.}& \left\{\begin{align}-6 x+15 y &=10 \\ 2 x-5 y &= -5 \end{aligned}\right. \\\\ \text{Умножьте второе уравнение на 3, чтобы исключить переменную.} & \left\{\begin{array}{l}{-6 x+15 y= 10} \\ {3(2 x-5 y)=3(-5)}\end{массив}\right. \\\\ \text{Упрощение и добавление.} & \left\{\begin{align} {-6 x+15 y =10} \\ \underline{6 x-15 y =-15} \end{выровнено}\right. \\ & \qquad \qquad \quad0\neq 5 \end{массив}\)

    Это утверждение неверно. Уравнения несовместимы, поэтому их графики будут параллельными линиями.

    Система не имеет решения.

    Упражнение \(\PageIndex{20}\)

    Решите систему методом исключения. \(\left\{\begin{array}{l}{-3 x+2 y=8} \\ {9 x-6 y=13}\end{array}\right.\)

    Ответить

    нет решения

    Упражнение \(\PageIndex{21}\)

    Решите систему методом исключения.\(\left\{\begin{array}{l}{7 x-3 y=-2} \\ {-14 x+6 y=8}\end{array}\right.\)

    Ответить

    нет решения

    Некоторые прикладные задачи переводятся непосредственно в уравнения в стандартной форме, поэтому для их решения мы будем использовать метод исключения. Как и прежде, мы используем нашу Стратегию решения проблем, чтобы оставаться сосредоточенными и организованными.

    Упражнение \(\PageIndex{22}\)

    Сумма двух чисел равна 39.Их разница равна 9. Найдите числа.

    Ответить

    \(\begin{array} {ll} \textbf{Шаг 1. Прочтите}\text{ проблему}& \\ \textbf{Шаг 2. Определите} \text{ что мы ищем.} & \text{ Ищем два числа.} \\\textbf{Шаг 3. Имя} \text{ то, что ищем.} & \text{Пусть n = первое число.} \\ & \text{ m = второе number} \\\textbf{Шаг 4. Преобразуйте} \text{ в систему уравнений.}& \\ & \text{Сумма двух чисел равна 39.} \\ & n+m=39\\ & \text{Их разница равна 9.} \\ & n−m=9 \\ \\ \text{Система:} & \left\{\begin{массив }{l}{n+m=39} \\ {nm=9}\end{массив}\right. \\\\ \textbf{Шаг 5. Решите} \text{ систему уравнений. } & \\ \text{Чтобы решить систему уравнений, используйте} \\ \text{исключение. Уравнения имеют стандартную} \\ \text{форму, а коэффициенты при m} & \\ \text{противоположны. Добавить.} & \left\{\begin{массив}}{l}{n+m=39} \\ \underline{nm=9}\end{массив}\right. \\ &\quad 2n\qquad=48 \\ \\\text{Найти n. } & n=24 \\ \\ \text{Подставить n=24 в одно из исходных} &n+m=39 \\ \text{уравнения и решить форму.} & 24+m=39 \\ & m=15 \\ \textbf{Шаг 6. Проверка}\text{ ответа.} & \text{Поскольку 24+15=39 и 24−15=9, ответы проверяются.}\\ \textbf{Шаг 7. Ответ} \ text{ вопрос.} & \text{Числа 24 и 15.} \end{массив}\)

    Упражнение \(\PageIndex{23}\)

    Сумма двух чисел равна 42. Их разница равна 8. Найдите числа.

    Ответить

    Цифры 25 и 17.

    Упражнение \(\PageIndex{24}\)

    Сумма двух чисел равна −15. Их разница составляет −35. Найдите числа.

    Ответить

    Числа -25 и 10.

    Упражнение \(\PageIndex{25}\)

    Каждый день по пути на работу Джо останавливается в бургерной. В понедельник он заказал один средний картофель фри и две маленькие газированные напитки, в которых было в общей сложности 620 калорий. Во вторник у него было два заказа картофеля фри среднего размера и одна маленькая газировка, всего 820 калорий. Сколько калорий в одном заказе картофеля фри среднего размера? Сколько калорий в одной маленькой газировке?

    Ответить

    Упражнение \(\PageIndex{26}\) ​​

    Малик останавливается в продуктовом магазине, чтобы купить пакет подгузников и 2 банки детских смесей. Всего он тратит 37 долларов. На следующей неделе он останавливается и покупает 2 пакета подгузников и 5 банок со смесью на общую сумму 87 долларов.Сколько стоит мешок подгузников? Сколько стоит банка смеси?

    Ответить

    Пакет с подгузниками стоит 11 долларов, а банка со смесью — 13 долларов.

    Упражнение \(\PageIndex{27}\)

    Чтобы получить дневную норму фруктов, Саша съедает в среду банан и 8 ягод клубники, что составляет 145 калорий. В следующую среду она съедает два банана и 5 ягод клубники, что в сумме дает 235 калорий на фрукты. Сколько калорий в банане? Сколько калорий в клубнике?

    Ответить

    В банане 105 калорий, а в клубнике 5 калорий.

    Когда вам нужно будет решить систему линейных уравнений на последнем уроке математики, вам обычно не говорят, какой метод использовать. Вам нужно будет принять это решение самостоятельно. Таким образом, вы захотите выбрать метод, который проще всего сделать и сводит к минимуму вероятность ошибок.

    Упражнение \(\PageIndex{28}\)

    Для каждой системы линейных уравнений решите, как удобнее решать ее подстановкой или исключением. Поясните свой ответ.

    1. \(\left\{\begin{array}{l}{3 x+8 y=40} \\ {7 x-4 y=-32}\end{array}\right.\)
    2. \(\left\{\begin{array}{l}{5 x+6 y=12} \\ {y=\frac{2}{3} x-1}\end{array}\right.\ )
    Ответить

    1. \(\left\{\begin{массив}}{l}{3 x+8 y=40} \\ {7 x-4 y=-32}\end{массив}\right. \)

    Так как оба уравнения имеют стандартную форму, наиболее удобным будет использование метода исключения.

    2. \(\left\{\begin{array}{l}{5 x+6 y=12} \\ {y=\frac{2}{3} x-1}\end{array}\right .\)

    Так как одно уравнение уже решено для y , наиболее удобным будет использование подстановки.

    Упражнение \(\PageIndex{29}\)

    Для каждой системы линейных уравнений решите, будет ли удобнее решать ее подстановкой или исключением.Поясните свой ответ.

    1. \(\left\{\begin{array}{l}{4 x-5 y=-32} \\ {3 x+2 y=-1}\end{array}\right.\)
    2. \(\left\{\begin{массив}}{l}{x=2 y-1} \\ {3 x-5 y=-7}\end{массив}\right.\)
    Ответить
    1. Поскольку оба уравнения имеют стандартную форму, наиболее удобным будет использование исключения.
    2. Так как одно уравнение уже решено относительно xx, наиболее удобным будет использование подстановки.

    Упражнение \(\PageIndex{30}\)

    Для каждой системы линейных уравнений решите, будет ли удобнее решать ее подстановкой или исключением.Поясните свой ответ.

    1. \(\left\{\begin{массив}}{l}{y=2 x-1} \\ {3 x-4 y=-6}\end{массив}\right.\)
    2. \(\left\{\begin{массив}}{l}{6 x-2 y=12} \\ {3 x+7 y=-13}\end{массив}\right.\)
    Ответить
    1. Так как одно уравнение уже решено относительно yy, наиболее удобным будет использование подстановки;
    2. Поскольку оба уравнения имеют стандартную форму, наиболее удобным будет использование метода исключения.

    Примечание

    Получите доступ к этим онлайн-ресурсам для получения дополнительных инструкций и практики решения систем линейных уравнений методом исключения.

    • Решение системы уравнений методом исключения
      1. Запишите оба уравнения в стандартной форме. Если какие-либо коэффициенты являются дробями, очистите их.
      2. Сделайте коэффициенты одной переменной противоположными.
        • Решите, какую переменную вы удалите.
        • Умножьте одно или оба уравнения так, чтобы коэффициенты этой переменной были противоположны.
      3. Добавьте уравнения, полученные на шаге 2, чтобы исключить одну переменную.
      4. Найдите оставшуюся переменную.
      5. Подставьте решение шага 4 в одно из исходных уравнений. Затем найдите другую переменную.
      6. Запишите решение в виде упорядоченной пары.
      7. Убедитесь, что упорядоченная пара является решением обоих исходных уравнений.

    Практика ведет к совершенству

    Решение системы уравнений методом исключения

    В следующих упражнениях решите системы уравнений методом исключения.

    Упражнение \(\PageIndex{1}\)

    \(\left\{\begin{массив}}{l}{5 x+2 y=2} \\ {-3 xy=0}\end{массив}\right.\)

    Упражнение \(\PageIndex{2}\)

    \(\left\{\begin{array}{l}{-3 x+y=-9} \\ {x-2 y=-12}\end{array}\right.\)

    Ответить

    \((6,9)\)

    Упражнение \(\PageIndex{3}\)

    \(\left\{\begin{array}}{l}{6 x-5 y=-1} \\ {2 x+y=13}\end{array}\right.\)

    Упражнение \(\PageIndex{4}\)

    \(\left\{\begin{массив}}{l}{3 xy=-7} \\ {4 x+2 y=-6}\end{массив}\right.\)

    Ответить

    \((-2,1)\)

    Упражнение \(\PageIndex{5}\)

    \(\left\{\begin{массив}}{l}{x+y=-1} \\ {xy=-5}\end{массив}\right.\)

    Упражнение \(\PageIndex{6}\)

    \(\left\{\begin{массив}}{l}{x+y=-8} \\ {xy=-6}\end{массив}\right.\)

    Ответить

    \((-7,-1)\)

    Упражнение \(\PageIndex{7}\)

    \(\left\{\begin{массив}}{l}{3 x-2 y=1} \\ {-x+2 y=9}\end{массив}\right. \)

    Упражнение \(\PageIndex{8}\)

    \(\left\{\begin{array}{l}{-7 x+6 y=-10} \\ {x-6 y=22}\end{array}\right.\)

    Ответить

    \((-2,-4)\)

    Упражнение \(\PageIndex{9}\)

    \(\left\{\begin{array}{l}{3 x+2 y=-3} \\ {-x-2 y=-19}\end{array}\right.\)

    Упражнение \(\PageIndex{10}\)

    \(\left\{\begin{массив}}{l}{5 x+2 y=1} \\ {-5 x-4 y=-7}\end{массив}\right.\)

    Ответить

    \((-1,3)\)

    Упражнение \(\PageIndex{11}\)

    \(\left\{\begin{array}{l}{6 x+4 y=-4} \\ {-6 x-5 y=8}\end{array}\right.\)

    Упражнение \(\PageIndex{12}\)

    \(\left\{\begin{array}{l}{3 x-4 y=-11} \\ {x-2 y=-5}\end{array}\right.\)

    Ответить

    \((-1,2)\)

    Упражнение \(\PageIndex{13}\)

    \(\left\{\begin{массив}}{l}{5 x-7 y=29} \\ {x+3 y=-3}\end{массив}\right. \)

    Упражнение \(\PageIndex{14}\)

    \(\left\{\begin{array}{l}{6 x-5 y=-75} \\ {-x-2 y=-13}\end{array}\right.\)

    Ответить

    \((-5,9)\)

    Упражнение \(\PageIndex{15}\)

    \(\left\{\begin{array}}{l}{-x+4 y=8} \\ {3 x+5 y=10}\end{array}\right.\)

    Упражнение \(\PageIndex{16}\)

    \(\left\{\begin{массив}}{l}{2 x-5 y=7} \\ {3 xy=17}\end{массив}\right.\)

    Ответить

    \((6,1)\)

    Упражнение \(\PageIndex{17}\)

    \(\left\{\begin{массив}}{l}{5 x-3 y=-1} \\ {2 xy=2}\end{массив}\right.\)

    Упражнение \(\PageIndex{18}\)

    \(\left\{\begin{массив}}{l}{7 x+y=-4} \\ {13 x+3 y=4}\end{массив}\right.\)

    Ответить

    \((-2,10)\)

    Упражнение \(\PageIndex{19}\)

    \(\left\{\begin{массив}}{l}{-3 x+5 y=-13} \\ {2 x+y=-26}\end{массив}\right. \)

    Упражнение \(\PageIndex{20}\)

    \(\left\{\begin{array}}{l}{3 x-5 y=-9} \\ {5 x+2 y=16}\end{array}\right.\)

    Ответить

    \((2,3)\)

    Упражнение \(\PageIndex{21}\)

    \(\left\{\begin{массив}}{l}{4 x-3 y=3} \\ {2 x+5 y=-31}\end{массив}\right.\)

    Упражнение \(\PageIndex{22}\)

    \(\left\{\begin{массив}}{l}{4 x+7 y=14} \\ {-2 x+3 y=32}\end{массив}\right.\)

    Ответить

    \((-7,6)\)

    Упражнение \(\PageIndex{23}\)

    \(\left\{\begin{массив}}{l}{5 x+2 y=21} \\ {7 x-4 y=9}\end{массив}\right.\)

    Упражнение \(\PageIndex{24}\)

    \(\left\{\begin{массив}}{l}{3 x+8 y=-3} \\ {2 x+5 y=-3}\end{массив}\right.\)

    Ответить

    \((-9,3)\)

    Упражнение \(\PageIndex{25}\)

    \(\left\{\begin{массив}}{l}{11 x+9 y=-5} \\ {7 x+5 y=-1}\end{массив}\right. \)

    Упражнение \(\PageIndex{26}\) ​​

    \(\left\{\begin{array}}{l}{3 x+8 y=67} \\ {5 x+3 y=60}\end{array}\right.\)

    Ответить

    \((9,5)\)

    Упражнение \(\PageIndex{27}\)

    \(\left\{\begin{массив}}{l}{2 x+9 y=-4} \\ {3 x+13 y=-7}\end{массив}\right.\)

    Упражнение \(\PageIndex{28}\)

    \(\left\{\begin{array}{l}{\frac{1}{3} xy=-3} \\ {x+\frac{5}{2} y=2}\end{array} \правильно.\)

    Ответить

    \((-3,2)\)

    Упражнение \(\PageIndex{29}\)

    \(\left\{\begin{array}{l}{x+\frac{1}{2} y=\frac{3}{2}} \\ {\frac{1}{5} x-\ frac{1}{5} y=3}\end{массив}\right.\)

    Упражнение \(\PageIndex{30}\)

    \(\left\{\begin{array}{l}{x+\frac{1}{3} y=-1} \\ {\frac{1}{2} x-\frac{1}{3 } y=-2}\end{массив}\right.\)

    Ответить

    \((-2,3)\)

    Упражнение \(\PageIndex{31}\)

    \(\left\{\begin{array}{l}{\frac{1}{3} xy=-3} \\ {\frac{2}{3} x+\frac{5}{2} y =3}\конец{массив}\справа. \)

    Упражнение \(\PageIndex{32}\)

    \(\left\{\begin{array}}{l}{2 x+y=3} \\ {6 x+3 y=9}\end{array}\right.\)

    Ответить

    бесконечно много решений

    Упражнение \(\PageIndex{33}\)

    \(\left\{\begin{array}{l}{x-4 y=-1} \\ {-3 x+12 y=3}\end{array}\right.\)

    Упражнение \(\PageIndex{34}\)

    \(\left\{\begin{массив}}{l}{-3 xy=8} \\ {6 x+2 y=-16}\end{массив}\right.\)

    Ответить

    бесконечно много решений

    Упражнение \(\PageIndex{35}\)

    \(\left\{\begin{массив}}{l}{4 x+3 y=2} \\ {20 x+15 y=10}\end{массив}\right.\)

    Упражнение \(\PageIndex{36}\)

    \(\left\{\begin{array}{l}{3 x+2 y=6} \\ {-6 x-4 y=-12}\end{array}\right.\)

    Ответить

    бесконечно много решений

    Упражнение \(\PageIndex{37}\)

    \(\left\{\begin{массив}}{l}{5 x-8 y=12} \\ {10 x-16 y=20}\end{массив}\right. \)

    Упражнение \(\PageIndex{38}\)

    \(\left\{\begin{array}{l}{-11 x+12 y=60} \\ {-22 x+24 y=90}\end{array}\right.\)

    Ответить

    противоречиво, нет решения

    Упражнение \(\PageIndex{39}\)

    \(\left\{\begin{array}{l}{7 x-9 y=16} \\ {-21 x+27 y=-24}\end{array}\right.\)

    Упражнение \(\PageIndex{40}\)

    \(\left\{\begin{array}{l}{5 x-3 y=15} \\ {y=\frac{5}{3} x-2}\end{массив}\right.\)

    Ответить

    противоречиво, нет решения

    Упражнение \(\PageIndex{41}\)

    \(\left\{\begin{array}{l}{2 x+4 y=7} \\ {y=-\frac{1}{2} x-4}\end{массив}\right. \)

    Решение приложений систем уравнений методом исключения

    В следующих упражнениях переведите в систему уравнений и решите.

    Упражнение \(\PageIndex{42}\)

    Сумма двух чисел равна 65. Их разница 25. Найдите числа.

    Ответить

    Цифры 20 и 45.

    Упражнение \(\PageIndex{43}\)

    Сумма двух чисел равна 37. Их разница равна 9. Найдите числа.

    Упражнение \(\PageIndex{44}\)

    Сумма двух чисел равна −27. Их разница составляет −59. Найдите числа.

    Ответить

    Числа 16 и -43.

    Упражнение \(\PageIndex{45}\)

    Сумма двух чисел равна −45. Их разница составляет −89. Найдите числа.

    Упражнение \(\PageIndex{46}\)

    Андреа покупает новые рубашки и свитера. Она может купить 3 рубашки и 2 свитера за 114 долларов или 2 рубашки и 4 свитера за 164 доллара. Сколько стоит рубашка? Сколько стоит свитер?

    Ответить

    Рубашка стоит 16 долларов, а свитер — 33 доллара.

    Упражнение \(\PageIndex{47}\)

    Питер покупает канцтовары. Он может купить 3 упаковки бумаги и 4 степлера за 40 долларов или 5 упаковок бумаги и 6 степлеров за 62 доллара. Сколько стоит пачка бумаги? Сколько стоит степлер?

    Упражнение \(\PageIndex{48}\)

    Общее количество натрия в 2 хот-догах и 3 чашках творога составляет 4720 мг. Общее количество натрия в 5 хот-догах и 2 чашках творога составляет 6300 мг.Сколько натрия в хот-доге? Сколько натрия в чашке творога?

    Ответить

    В хот-доге 860 мг. В чашке творога 1000 мг.

    Упражнение \(\PageIndex{49}\)

    Общее количество калорий в 2 хот-догах и 3 чашках творога составляет 960 калорий. Общее количество калорий в 5 хот-догах и 2 чашках творога составляет 1190 калорий. Сколько калорий в хот-доге? Сколько калорий в чашке творога?

    Выберите наиболее удобный метод решения системы линейных уравнений

    В следующих упражнениях решите, будет ли удобнее решать систему уравнений путем замены или исключения.

    Упражнение \(\PageIndex{50}\)

    1. \( \left\{\begin{массив}}{l}{8 x-15 y=-32} \\ {6 x+3 y=-5}\end{массив}\right.\)
    2. \(\left\{\begin{массив}}{l}{x=4 y-3} \\ {4x-2 y=-6}\end{массив}\right.\)
    Ответить
    1. устранение
    2. замена

    Упражнение \(\PageIndex{51}\)

    1. \(\left\{\begin{массив}}{l}{y=7 x-5} \\ {3 x-2 y=16}\end{массив}\right.\)
    2. \(\left\{\begin{array}{l}{12 x-5 y=-42} \\ {3 x+7 y=-15}\end{array}\right.\)

    Упражнение \(\PageIndex{52}\)

    1. \(\left\{\begin{array}{l}{y=4 x+9} \\ {5 x-2 y=-21}\end{array}\right.\)
    2. \(\left\{\begin{array}{l}{9 x-4 y=24} \\ {3 x+5 y=-14}\end{array}\right.\)
    Ответить
    1. замена
    2. устранение

    Упражнение \(\PageIndex{53}\)

    1. \(\left\{\begin{массив}}{l}{14 x-15 y=-30} \\ {7 x+2 y=10}\end{массив}\right. \)
    2. \(\left\{\begin{array}{l}{x=9 y-11} \\ {2x-7 y=-27}\end{array}\right.\)

    Математика на каждый день

    Упражнение \(\PageIndex{54}\)

    Норрис может прогрести 3 мили вверх по течению за то же время, что и 5 миль вниз по течению по течению. Решите систему. \(\left\{\begin{массив}}{l}{rc=3} \\ {r+c=5}\end{массив}\right.\)

    1. для r, его скорость гребли в стоячей воде.
    2. Затем найдите с, скорость течения реки.
    Ответить
    1. г=4
    2. с=1

    Упражнение \(\PageIndex{55}\)

    Джози хочет приготовить 10 фунтов смеси из орехов и изюма, и она хочет, чтобы общая стоимость смеси составила 54 доллара. Орехи стоят 6 долларов за фунт, а изюм — 3 доллара за фунт. Решите систему \(\left\{\begin{array}{l}{n+r=10} \\ {6 n+3 r=54}\end{array}\right.\), чтобы найти n, количество фунтов орехов и rr — количество фунтов изюма, которое она должна использовать.

    Письменные упражнения

    Упражнение \(\PageIndex{56}\)

    Решите систему
    \(\left\{\begin{array}{l}{x+y=10} \\ {5 x+8 y=56}\end{array}\right.\)

    1. путем замены
    2. путем построения графика
    3. Какой метод вы предпочитаете? Почему?
    Ответить
    1. (8, 2)

    3. Ответы будут разными.

    Упражнение \(\PageIndex{57}\)

    Решить систему \(\left\{\begin{array}{l}{x+y=-12} \\ {y=4-\frac{1}{2} x}\end{array}\right .\)

    1. путем замены
    2. путем построения графика
    3. Какой метод вы предпочитаете? Почему?

    Самопроверка

    ⓐ После выполнения упражнений используйте этот контрольный список, чтобы оценить свое мастерство выполнения целей этого раздела.

    ⓑ Что этот контрольный список говорит вам о вашем мастерстве в этом разделе? Какие шаги вы предпримете для улучшения?

    систем линейных уравнений, примеры решений, картинки и практические задачи.

    Система просто ..
    Что такое система уравнений?
    Отвечать

    Система уравнений просто означает «более 1 уравнения». Система линейных уравнений — это чуть больше 1 строки, см. рисунок:

    Итак, что такое
    решение системы уравнений? Отвечать

    Решение находится там, где уравнения «встречаются» или пересекаются.Красная точка — решение системы.

    Сколько решений может иметь система линейных уравнений?
    Отвечать

    Могут быть нулевые решения, 1 решение или бесконечные решения — каждый случай подробно объясняется ниже. Примечание. Хотя системы линейных уравнений могут состоять из 3 и более уравнений, мы будем ссылаться на наиболее распространенный случай — основу, содержащую ровно 2 линии.

    Случай I: 1 Решение

    Это наиболее распространенная ситуация, когда линии пересекаются ровно в одной точке.


    Случай 2: Нет решений

    Это происходит только тогда, когда прямые параллельны. Как видите, параллельные линии никогда не пересекутся.

    Пример стебля, не имеющего решения:

    • Строка 1: $$ y = 5x + 13 $$
    • Строка 2: $$ y = 5x + 12 $$

    Дело III: Бесконечные решения

    Это самый редкий случай и происходит только при наличии той же строки
    Рассмотрим, например, две строки ниже (y = 2x + 1 и 2y = 4x + 2).Эти два уравнения на самом деле представляют собой одну и ту же прямую.

    Пример системы с бесконечными решениями:

    • Строка 1: y = 2x + 1
    • Строка 2: 2y = 4x + 2
    Как мы можем найти решения систем уравнений?

    Найти решение системы линейных уравнений можно любым из способов, указанных ниже:

    Решение систем уравнений подстановкой — Элементарная алгебра

    Цели обучения

    К концу этого раздела вы сможете:

    • Решить систему уравнений подстановкой
    • Решить приложения систем уравнений подстановкой

    Решение систем линейных уравнений с помощью графика — хороший способ визуализировать типы решений, которые могут получиться. Однако во многих случаях решение системы с помощью графика неудобно или неточно. Если графики выходят за пределы небольшой сетки с размерами x и y в диапазоне от -10 до 10, построение линий может оказаться громоздким. И если решения системы не являются целыми числами, может быть трудно точно прочитать их значения по графику.

    В этом разделе мы будем решать системы линейных уравнений методом подстановки.

    Решение системы уравнений путем замены

    Мы будем использовать ту же систему, которую использовали для построения графиков.

    Сначала мы решим одно из уравнений либо для x , либо для y . Мы можем выбрать любое уравнение и найти решение для любой переменной, но мы постараемся сделать такой выбор, который облегчит нам работу.

    Затем мы подставляем это выражение в другое уравнение. В результате получится уравнение только с одной переменной, и мы знаем, как его решить!

    После того, как мы найдем значение одной переменной, мы подставим это значение в одно из исходных уравнений и решим для другой переменной. Наконец, мы проверяем наше решение и убеждаемся, что оба уравнения верны.

    Теперь мы заполним все эти шаги (Рисунок).

    Как решить систему уравнений путем замены

    Решите систему подстановкой.

     

     

     

    Решите систему подстановкой.

    Решите систему подстановкой.

    Решите систему уравнений путем замены.

    1. Решите одно из уравнений для любой переменной.
    2. Подставьте выражение из шага 1 в другое уравнение.
    3. Решите полученное уравнение.
    4. Подставьте решение шага 3 в одно из исходных уравнений, чтобы найти другую переменную.
    5. Запишите решение в виде упорядоченной пары.
    6. Убедитесь, что упорядоченная пара является решением обоих исходных уравнений.

    Если одно из уравнений в системе задано в виде наклона и точки пересечения, шаг 1 уже выполнен! Мы увидим это на (рис. ).

    Решите систему подстановкой.

    Решение

    Второе уравнение уже решено для y . Подставим выражение вместо y в первое уравнение.

    Решите систему подстановкой.

    Решите систему подстановкой.

    Если уравнения даны в стандартной форме, нам нужно начать с решения для одной из переменных.В следующем примере мы решим первое уравнение для y .

    Решите систему подстановкой.

    Решение

    Нам нужно решить одно уравнение для одной переменной. Затем мы подставим это выражение в другое уравнение.

    Решите систему подстановкой.

    Решите систему подстановкой.

    На (рисунке) проще всего было найти y в первом уравнении, потому что оно имело коэффициент 1.На (Рисунок) будет проще решить для x .

    Решите систему подстановкой.

    Решение

    Решим первое уравнение, а затем подставим это выражение во второе уравнение.

    Решите систему подстановкой.

    Решите систему подстановкой.

    Когда оба уравнения уже решены для одной и той же переменной, их легко заменить!

    Решите систему подстановкой.

    Решение

    Поскольку оба уравнения решаются относительно y , мы можем подставить одно в другое.

    Решите систему подстановкой.

    Решите систему подстановкой.

    Будьте очень осторожны со знаками в следующем примере.

    Решите систему подстановкой.

    Решение

    Нам нужно решить одно уравнение для одной переменной. Решим первое уравнение для y .

    Решите систему подстановкой.

    Решите систему подстановкой.

    В (Рисунок) потребуется немного больше усилий, чтобы решить одно уравнение для x или y .

    Решите систему подстановкой.

    Решение

    Нам нужно решить одно уравнение для одной переменной. Мы решим первое уравнение для x .

    Поскольку 0 = 0 является истинным утверждением, система непротиворечива.Уравнения зависимы. Графики этих двух уравнений дадут одну и ту же линию. Система имеет бесконечно много решений.

    Решите систему подстановкой.

    бесконечно много решений

    Решите систему подстановкой.

    бесконечно много решений

    Посмотрите еще раз на уравнения на (Рисунок). Есть ли способ узнать, что это одна и та же линия?

    Давайте посмотрим, что произойдет в следующем примере.

    Решите систему подстановкой.

    Решение

    Второе уравнение уже решено для y , поэтому мы можем заменить y в первом уравнении.

    Поскольку 0 = −10 — ложное утверждение, уравнения несовместимы. Графики двух уравнений будут параллельными линиями. Система не имеет решений.

    Решите систему подстановкой.

    Решите систему подстановкой.

    Решение приложений систем уравнений путем замены

    Мы скопируем сюда стратегию решения задач, которую мы использовали в разделе «Решение систем уравнений с помощью графика» для решения систем уравнений.Теперь, когда мы знаем, как решать системы подстановкой, этим мы и займемся на шаге 5.

    Как использовать стратегию решения задач для систем линейных уравнений.

    1. Прочтите проблему. Убедитесь, что все слова и идеи понятны.
    2. Определите , что мы ищем.
    3. Имя то что мы ищем. Выберите переменные для представления этих величин.
    4. Переведите число в систему уравнений.
    5. Решите систему уравнений, используя хорошие методы алгебры.
    6. Проверьте ответ в задаче и убедитесь, что он имеет смысл.
    7. Ответьте на вопрос полным предложением.

    Некоторым людям проще решать текстовые задачи с двумя переменными, чем с одной переменной. Выбирать имена переменных проще, когда все, что вам нужно сделать, это написать две буквы. Подумайте об этом в следующем примере — как бы вы сделали это, используя всего одну переменную?

    Сумма двух чисел равна нулю.Одно число на девять меньше другого. Найдите числа.

    Сумма двух чисел равна 10. Одно число на 4 меньше другого. Найдите числа.

    Сумма двух чисел равна −6. Одно число на 10 меньше другого. Найдите числа.

    Числа 2 и −8.

    На (Рисунке) мы будем использовать формулу для периметра прямоугольника, P = 2 L + 2 W .

    Периметр прямоугольника равен 88.Длина в пять раз больше ширины. Найдите длину и ширину.

    Периметр прямоугольника равен 40. Длина в 4 раза больше ширины. Найдите длину и ширину прямоугольника.

    Длина 12 и ширина 8.

    Периметр прямоугольника равен 58. Длина в 5 раз больше ширины. Найдите длину и ширину прямоугольника.

    Длина 23 и ширина 6.

    Для (Рисунок) нам нужно помнить, что сумма мер углов треугольника составляет 180 градусов и что прямоугольный треугольник имеет один угол 90 градусов.

    Размер одного из малых углов прямоугольного треугольника в десять раз больше, чем размер другого малого угла. Найдите меры обоих углов.

    Решение

    Нарисуем и подпишем фигуру.

    Один из малых углов прямоугольного треугольника в 2 и более раза больше другого малого угла. Найдите величину обоих углов.

    Мера углов 22 градуса и 68 градусов.

    Размер одного из малых углов прямоугольного треугольника на 18 меньше, чем удвоенный размер другого малого угла. Найдите величину обоих углов.

    Мера углов 36 градусов и 54 градуса.

    Хизер предложили два варианта оплаты труда тренера в тренажерном зале. Вариант А будет платить ей 25 000 фунтов стерлингов плюс 15 фунтов стерлингов за каждую тренировку. Вариант Б будет платить ей 10 000 евро + 40 фунтов стерлингов за каждую тренировку. Сколько учебных занятий уравняло бы варианты заработной платы?

    Две страховые компании предложили Джеральдин работу.Первая компания платит зарплату в размере 12 000 фунтов стерлингов плюс комиссионные в размере 100 фунтов стерлингов за каждый проданный полис. Второй платит зарплату в размере 20 000 фунтов стерлингов плюс комиссионные в размере 50 фунтов стерлингов за каждый проданный полис. Сколько полисов нужно продать, чтобы общая сумма выплат осталась прежней?

    Потребуется продать 160 полисов, чтобы общая сумма выплат осталась прежней.

    В настоящее время Кеннет продает костюмы для компании А, получая зарплату в размере 22 000 фунтов стерлингов плюс комиссионные в размере 10 фунтов стерлингов за каждый проданный костюм. Компания B предлагает ему должность с окладом в 28 000 фунтов стерлингов плюс комиссионные в размере 4 евро за каждый проданный костюм.Сколько костюмов нужно продать Кеннету, чтобы варианты были равны?

    Кеннету нужно продать 1000 костюмов.

    Ключевые понятия

    • Решите систему уравнений подстановкой
      1. Решите одно из уравнений для любой переменной.
      2. Подставьте выражение из шага 1 в другое уравнение.
      3. Решите полученное уравнение.
      4. Подставьте решение шага 3 в одно из исходных уравнений, чтобы найти другую переменную.
      5. Запишите решение в виде упорядоченной пары.
      6. Убедитесь, что упорядоченная пара является решением обоих исходных уравнений.
    Практика делает совершенным

    Решение системы уравнений путем замены

    В следующих упражнениях решите системы уравнений путем замены.

    Бесконечное множество решений

    Бесконечное множество решений

    Решение приложений систем уравнений путем замены

    В следующих упражнениях переведите в систему уравнений и решите.

    Сумма двух чисел равна 15. Одно число на 3 меньше другого. Найдите числа.

    Сумма двух чисел равна 30. Одно число на 4 меньше другого. Найдите числа.

    Сумма двух чисел равна −26. Одно число на 12 меньше другого. Найдите числа.

    Числа -7 и -19.

    Периметр прямоугольника равен 50. Длина в 5 раз больше ширины. Найдите длину и ширину.

    Периметр прямоугольника равен 60.Длина в 10 раз больше ширины. Найдите длину и ширину.

    Длина 20 и ширина 10.

    Периметр прямоугольника равен 58. Длина в 5 раз больше ширины. Найдите длину и ширину.

    Периметр прямоугольника равен 84. Длина в 10 раз больше ширины, чем в три раза. Найдите длину и ширину.

    Длина 34 и ширина 8.

    Размер одного из малых углов прямоугольного треугольника в 14 раз больше, чем размер другого малого угла.Найдите величину обоих углов.

    Размер одного из малых углов прямоугольного треугольника в 26 раз больше, чем размер другого малого угла. Найдите величину обоих углов.

    Размеры 16° и 74°.

    Размер одного из малых углов прямоугольного треугольника на 15 меньше, чем удвоенный размер другого малого угла. Найдите величину обоих углов.

    Размер одного из малых углов прямоугольного треугольника на 45 меньше, чем удвоенный размер другого малого угла.Найдите величину обоих углов.

    Размеры: 45° и 45°.

    Максиму предложили работу два автодилера. Первая компания платит зарплату в размере 10 000 евро плюс комиссионные в размере 1 000 евро за каждую проданную машину. Второй получает зарплату в размере 20 000 фунтов стерлингов плюс комиссионные в размере 500 фунтов стерлингов за каждую проданную машину. Сколько автомобилей нужно продать, чтобы общая сумма выплат осталась прежней?

    Две кабельные компании предложили Джеки вакансии. Первая компания платит зарплату ? 14 000 плюс комиссия в размере 100 фунтов стерлингов за каждый проданный кабельный пакет.Второй получает зарплату в размере 20 000 фунтов стерлингов плюс комиссионные в размере 25 фунтов стерлингов за каждый проданный кабельный пакет. Сколько пакетов кабельного телевидения необходимо продать, чтобы общая сумма выплат осталась прежней?

    Необходимо продать 80 пакетов кабелей.

    В настоящее время Амара продает телевизоры для компании А, получая зарплату в размере 17 000 фунтов стерлингов плюс комиссионные в размере 100 фунтов стерлингов за каждый телевизор, который она продает. Компания B предлагает ей должность с окладом в 29 000 фунтов стерлингов плюс комиссионные в размере 20 фунтов стерлингов за каждый проданный ею телевизор. Сколько телевизоров Амара должна продавать, чтобы варианты были равными?

    В настоящее время Митчелл продает печи для компании А, получая зарплату в размере 12 000 фунтов стерлингов плюс комиссионные в размере 150 фунтов стерлингов за каждую проданную им печь.Компания B предлагает ему должность с окладом в 24 000 фунтов стерлингов плюс комиссионные в размере 50 фунтов стерлингов за каждую проданную им печь. Сколько печей нужно продать Митчеллу, чтобы варианты были равными?

    Митчеллу нужно продать 120 печей.

    Математика на каждый день

    Когда Глория провела 15 минут на эллиптическом тренажере, а затем занималась круговой тренировкой в ​​течение 30 минут, ее фитнес-приложение сообщает, что она сожгла 435 калорий. Когда она провела 30 минут на эллиптическом тренажере и 40 минут круговой тренировки, она сожгла 690 калорий.Решите систему для , количества калорий, которые она сжигает за каждую минуту на эллиптическом тренажере, и , количества калорий, которые она сжигает за каждую минуту круговой тренировки.

    Письменные упражнения

    Решить систему уравнений

    ⓐ графически.
    ⓑ путем замены.
    ⓒ Какой метод вы предпочитаете? Почему?

    Решите систему уравнений
    подстановкой и объясните все свои действия словами.

    Самопроверка

    ⓐ После выполнения упражнений используйте этот контрольный список, чтобы оценить свое мастерство выполнения целей этого раздела.

    ⓑ Изучив этот контрольный список, что вы сделаете, чтобы стать уверенным в выполнении всех задач?

    Системы линейных уравнений: построение графиков

    Пурпурная математика

    Когда вы решаете системы уравнений (линейных или других), вы, с точки зрения графических линий, связанных с уравнениями, находите любые точки пересечения этих линий.

    Если система уравнений состоит только из пары линейных уравнений с двумя переменными, то уравнение этой системы можно изобразить графически; график будет содержать две прямые линии, а решением системы будет точка (точки) пересечения этих линий. Поскольку две прямые линии на плоскости могут быть построены только тремя способами, то для данной системы уравнений существует только три соответствующих формы решения.

    Справка по математике.

    ком

    Две прямые линии (1) имеют разные наклоны и точки пересечения, поэтому они пересекаются ровно в одной точке, (2) параллельны с разными точками пересечения, поэтому никогда не пересекаются ни в каких точках, или (3) имеют одинаковый наклон и точки пересечения, так что они на самом деле являются одной и той же линией, поэтому они «пересекаются» везде (где «везде» означает «везде идет одна линия, туда идет и другая линия; у них есть все точки — бесконечно много точек — общие»).Эти три случая для пар прямых линий проиллюстрированы ниже:

    На первом графике выше, представляющем собой «Случай 1» в левом столбце, показаны две отдельные непараллельные линии, которые пересекаются ровно в одной точке. Соответствующая система уравнений называется «независимой» системой, а решением является одна ( x , y ) точка.

    На втором графике выше, обозначенном как «Случай 2» в средней колонке, показаны две отдельные параллельные линии.Поскольку параллельные прямые никогда не пересекаются, то и пересечения этих прямых быть не может; то есть для системы двух линейных уравнений, которая изображается в виде двух параллельных линий, не может быть решения. Это называется «несогласованной» системой уравнений.

    На третьем графике выше, обозначенном как «Случай 3» в правом столбце, отображается только одна линия. На самом деле, это одна и та же линия, проведенная дважды. Эти «две» линии, на самом деле являющиеся одной и той же линией, «пересекаются» (в техническом смысле) в каждой точке своей длины.Это означает, что каждая точка на прямой(ых) является решением системы. Это называется «зависимой» системой, а «решением» является вся линия.

    Это показывает, что система уравнений может иметь одно решение (определенная точка x , y ), не иметь решения вообще или иметь бесконечное решение (все решения уравнения). У вас никогда не будет системы с двумя линейными уравнениями и двумя переменными с двумя или более решениями; он всегда будет один, ни один или бесконечно много.


    Если это зависимая система, то почему «бесконечно» или «все точки» не является правильным ответом?

    Хотя решение зависимой системы линейных уравнений представляет собой бесконечный набор точек, «бесконечность» не является числом, «бесконечность» недостаточно ясна, а точки вне линии (линий) системы не являются решениями. Только те точки, которые действительно лежат на прямой, являются решениями системы. Таким образом, ваш ответ для типа системы будет «зависимым», но решением на самом деле будет уравнение линии, поскольку это уравнение неявно перечисляет все точки, которые решают систему.

    Например, если два уравнения в зависимой системе сводятся к y = − x  + 3, то вы должны представить решение как линейное уравнение или использовать парное обозначение координат; а именно ( x , — x  + 3).


    Филиал

    Решение с помощью графика

    Вероятно, первым методом решения систем уравнений, который вы увидите, будет «решение с помощью графика».Предупреждение: вы должны относиться к этим проблемам с недоверием. Единственный способ найти решение на графике: , ЕСЛИ , вы рисуете очень аккуратную систему координат, , ЕСЛИ , вы рисуете очень аккуратные линии, , ЕСЛИ , решение оказывается точкой с хорошими аккуратными целочисленными координатами, и IF линии не близки к параллельным.

    Например, если линии пересекаются под небольшим углом, практически невозможно сказать, где они пересекаются.

    И если точка пересечения не является аккуратной парой целых чисел, все ставки сняты. Я имею в виду, можете ли вы сказать, просто взглянув…

    …что показанный выше перекресток имеет координаты (-4,3, -0,95)? Нет? Тогда вы понимаете мою точку зрения.

    Положительным моментом является то, что они будут вынуждены давать вам красивые и аккуратные решения для «решения с помощью графика», вы сможете получить правильные ответы до тех пор, пока вы рисуете очень аккуратно .Например:

    • Решите следующую систему с помощью графика.

    2 x — 3 y = -2
    4 x + y = 24

    Я знаю, что мне нужен четкий график, так что я возьму линейку и начну. Сначала я решу каждое уравнение для » y =», чтобы легко построить график:

    2 x — 3 y = -2
    2 x + 2 = 3 y
    (2/3) x + (2/3) = y

    4 x + y = 24
    y = −4 x + 24

    Вторую линию будет легко изобразить, используя только наклон и точку пересечения, но для первой линии мне понадобится Т-образная диаграмма.

    Иногда вы заметите пересечение прямо на Т-диаграмме. Вы видите точку, которая находится в обоих уравнениях выше? Проверьте заштрихованную серым строку выше.

    Теперь, когда у меня есть несколько точек, я аккуратно возьму линейку и график и поищу пересечение:

    Даже если бы я не заметил точку пересечения на Т-диаграмме, я ее точно увижу на картинке.

    Решение

    : ( x , y ) = (5, 4)

    Если вы застряли на «решении с помощью графика», пожалуйста, ради бога, нарисуйте линии так, чтобы они действительно пересекались в точке решения. Не будь неряшливым!


    Вы можете использовать приведенный ниже виджет Mathway, чтобы попрактиковаться в решении с помощью графика (или пропустить виджет и перейти к следующей странице). Попробуйте введенное упражнение (щелкнув стрелку и выбрав «Решить с помощью графика» во всплывающем окне) или введите свое собственное упражнение. Затем сравните свой ответ с ответом Мэтуэя. (Или пропустить виджет и продолжить урок.)

    (Нажмите здесь, чтобы перейти непосредственно на сайт Mathway, если вы хотите проверить их программное обеспечение или получить дополнительную информацию.)


    URL-адрес: https://www.purplemath.com/modules/systlin2.htm

    Общее решение системы уравнений

    На занятиях по алгебре, если система уравнений имеет бесконечное множество решений, вы просто пишете «бесконечное множество решений» и переходите к следующей задаче. Однако, когда мы говорим «бесконечно много решений», происходит гораздо больше. В этой статье мы рассмотрим эту идею с общими решениями.

    реклама

    Содержание:

    1. Запись общего решения
    2. Нахождение конкретных решений на основе общего решения
    3. Краткое описание шагов

    Запись общего решения

    Во-первых, давайте рассмотрим, как записать общее решение данной системы уравнений.Для этого рассмотрим пример.

    Пример

    Найдите общее решение системы уравнений:

    \(
    \begin{array}{c}
    x_1 + 2x_2 + 8x_3 + 18x_4 = 11\\
    x_1 + x_2 + 5x_3 +11x_4 = 10\\
    \end{массив}\)

    Как и в любой системе уравнений, мы будем использовать расширенную матрицу и сокращение строк.

    \(
    \left[
    \begin{array}{cccc|c}
    1 & 2 & 8 & 18 & 11\\
    1 & 1 & 5 & 11 & 10\\
    \end{массив}
    \right ]
    \sim
    \left[
    \begin{array}{cccc|c}
    1 & 0 & 2 & 4 & 9\\
    0 & 1 & 3 & 7 & 1\\
    \end{массив}
    \справа]
    \)

    Теперь выпишите уравнения из этой сокращенной матрицы.

    \(
    \begin{array}{c}
    x_1 + 2x_3 + 4x_4 = 9\\
    x_2 + 3x_3 + 7x_4 = 1\\
    \end{массив}\)

    Обратите внимание, что в матрице начальные единицы (первая ненулевая запись в каждой строке) находятся в столбцах для \(x_1\) и \(x_2\).

    Найдите эти переменные.

    \(
    \begin{array}{c}
    x_1 = 9 – 2x_3 – 4x_4\\
    x_2 = 1 – 3x_3 – 7x_4\\
    \end{массив}\)

    Остальные переменные являются свободными переменными , что означает, что они могут принимать любое значение.Значения \(x_1\) и \(x_2\) основаны на значении этих двух переменных. В общем решении, вы хотите отметить это.

    Общее решение:

    \(
    \boxed{
    \begin{array}{l}
    x_1 = 9 – 2x_3 – 4x_4\\
    x_2 = 1 – 3x_3 – 7x_4\\
    x_3 \text{ свободен}\\
    x_4 \text{ свободен}\\
    \end{array}
    }
    \)

    Существует бесконечно много решений этой системы уравнений, использующих различные значения двух свободных переменных.

    Поиск конкретных решений

    Предположим, вы хотите привести пример конкретного решения приведенной выше системы уравнений. Их бесконечно много, поэтому у вас есть большой выбор! Вам просто нужно рассмотреть возможные значения свободных переменных.

    Пример решения

    Пусть:

    \(
    \begin{array}{l}
    x_3 = 0\\
    x_4 = 1\\
    \end{массив}
    \)

    Не было особой причины выбирать 0 и 1. Опять же, это будет работать для ЛЮБОГО значения, которое вы выберете для этих двух переменных.

    Используя эти значения, решение:

    \(
    \begin{array}{l}
    x_1 = 9 – 2x_3 – 4x_4 = 9 – 2(0) – 4(1)\\
    x_2 = 1 – 3x_3 – 7x_4 = 1 – 3(0) – 7 (1)\\
    x_3 = 0\\
    x_4 = 1\\
    \end{array}
    \rightarrow
    \boxed{
    \begin{array}{l}
    x_1 = 5\\
    x_2 = -6 \\
    x_3 = 0\\
    x_4 = 1\\
    \end{массив}
    }
    \)

    Вы можете проверить эти значения в исходной системе уравнений, чтобы убедиться:

    \(
    \begin{array}{l}
    x_1 + 2x_2 + 8x_3 + 18x_4 = 11\\
    x_1 + x_2 + 5x_3 +11x_4 = 10\\
    \end{array}
    \rightarrow
    \begin{array} {l}
    (5) + 2(-6) + 8(0) + 18(1) = 11 \text{ (true)}\\
    (5) + (-6) + 5(0) +11 (1) = 10 \text{ (true)}\\
    \end{массив}
    \)

    Поскольку оба уравнения верны для этих значений, мы знаем, что нашли одно из многих решений.